You are on page 1of 110

1 Biological molecules Answers

Test yourself on prior knowledge (page 1)


1 Respiration
2 Amino acids
3 An atom is the smallest particle that something can be broken down into and still retain the
original chemical properties. It consists of a nucleus with atoms surrounding it. A molecule is
made up of two or more atoms held together by strong chemical bonds.

Test yourself (page 7)


1 The diagram should show the same principles as Figure 1.5 in the Student’s Book.
2 Starch is a polymer of alpha glucose, whereas cellulose is a polymer of beta glucose; starch
consists of amylose, which is helical, and amylopectin, which is branched, whereas cellulose
forms long straight chains; the alpha glycosidic bonds in starch are a different shape from the
beta glycosidic bonds in cellulose.
3 As starch is insoluble it is not ‘washed out’ of the cell, and as it does not affect osmosis it means
the cell it is stored in will not take in water by osmosis.
4 Glycogen is insoluble so does not get ‘washed away’ and does not affect osmosis; it is branched,
so there are lots of ‘ends’ to release glucose when needed; it is compact so can store a lot of
glucose in a small place.
5 C12H22O11
6 Different amounts of amylose and amylopectin; different number of monomers/α-glucose
molecules/β-glucose residues.

Test yourself (page 10)


7 Triglycerides are made of three fatty acids joined to one glycerol molecule, so they are not large
molecules made of many repeated smaller monomers joined together.
8 Triglycerides have a higher proportion of hydrogen than carbohydrates, and a lower proportion
of oxygen.
9 Triglycerides have three fatty acids joined to a glycerol molecule, whereas phospholipids have
only two fatty acids joined to a glycerol molecule. In place of a third fatty acid there is a
phosphate group.

Activity: Fatty acids in milk (page 10)


1 These fatty acids have no C=C double bonds so they must be saturated.

© Pauline Lowrie and Mark Smith 2015


1 Biological molecules Answers

Total concentration/mg g-1

Vegan group Control group

Saturated fatty acids 325 497

Unsaturated fatty acids 657 466

3 Linolenic or linoleic, because they have more than one C=C double bond.
4 There are more polyunsaturated fatty acids in milk produced by the vegan group than the
control group, probably because the vegan group consume more polyunsaturated fatty acids in
their diet. Saturated fatty acids are found mainly in animal fats and unsaturated fatty acids are
found mainly in plant food sources

Test yourself (page 13)


10 20 × 20 × 20 / 203 / 8000.
11 Both involve the loss of a molecule of water/both involve condensation.
12 C / The bonds holding the tertiary structure are broken.

Activity (page 14)


1

Tube Benedict’s test Benedict’s Iodine test Emulsion test Biuret test for
for reducing test for non- for starch for lipids protein
sugars reducing
sugars
C Orange-red Test not Stayed Stayed clear Mauve
precipitate carried out yellowy-
formed brown
A Stayed blue Stayed blue Stayed Stayed clear Mauve
yellowy-
brown
B Stayed blue Orange-red Blue-black Milky-white Stayed blue
precipitate colour emulsion
formed formed

2 If a solution gives a positive reducing sugar test, it will also give a positive non-reducing sugar
test as boiling reducing sugar with acid will not remove it. Therefore, it is only worth doing a
non-reducing sugar test on a solution that has already tested negative for reducing sugars.
3 The protease enzyme in tube A will have digested the protein into amino acids after an hour,
and amino acids do not give a positive biuret test for protein. However, the protease enzyme

© Pauline Lowrie and Mark Smith 2015


1 Biological molecules Answers

itself is a protein, and that will give a positive biuret test for protein. Tube C also gave a positive
result with the biuret test because it contained protein.
4 The student should take equal volumes of each solution, add equal volumes of Benedict’s
reagent to each, and boil each solution for the same amount of time. The stronger the orange-
red colour of the precipitate formed the more glucose is present in each tube.

Activity (page 15)


1 1 g of starch dissolved in distilled water and made up to 100 cm3, or 10 g starch dissolved in
distilled water and made up to 1 dm3.
2 0.0001%
3 Different types of starch contain different proportions of amylose and amylopectin.

Activity: Finding the concentration of reducing sugar in a


solution (page 16)
1 Add 10 cm3 of the 1 mol dm–3 solution to 100 cm3 distilled water to produce 0.1 mol dm–3
solution. Then take 10 cm3 of the 0.1 mol dm–3 solution and add this to 100 cm3 of distilled
water to make a 0.01 mol dm–3 solution. Continue until she has made a 0.0001 mol dm–3
solution.
2 Copper sulfate is blue, and the paler the blue of the solution, the more reducing sugar is
present. Red is the complementary colour to blue, so a blue solution will absorb more red light
than any other wavelength.
3 This means that light that is absorbed by the plastic cuvette and the water is accounted for. The
blank is the same as the test solutions except for the Benedict’s reagent and the glucose, so any
differences in % transmission will be the result of glucose and the Benedict’s reagent.
4 Plot % transmission of light on the y-axis and concentration of glucose solution on the x-axis,
and then join the points to produce a calibration curve.
5 A thicker cuvette blocks more light, so it would give a lower % transmission than a thinner
cuvette. It is important that cuvettes are very similar in thickness to ensure accurate results.
6 Carry out the method in the same way as before, and then measure the % transmission of red
light through each solution. She would find the % transmission on the y-axis of the graph, draw
a horizontal line across to the line on the graph, then a vertical line downwards. Where the line
meets the x-axis is the concentration of glucose in the solution.

© Pauline Lowrie and Mark Smith 2015


1 Biological molecules Answers

Practice questions (page 17)


1 a)

Disaccharide Formed from (Reject answer if an


incorrect monosaccharide is
Lactose Glucose and galactose [1] given in addition to a correct
one)
Maltose Glucose [1]

Sucrose Glucose and fructose [1]

b) Do reducing sugar test first and observe negative result;


Boil with (dilute) acid; Neutralise; Add Benedict’s reagent; Heat/boil; Orange/red precipitate
formed; (max. 4)

2 a) i) So that the two groups are as similar as possible; aspartame is the only variable/so there
is only one variable; [2]
ii) Pull names out of hat/throw a dice/use a computer/use random number table; [1]
(Accept an alternative as long as it would result in random distribution)
iii) To avoid bias; In reporting symptoms; [2]
(Do not allow unqualified references to psychological effects)
b) i)

(two molecules joined correctly and water removed for 2 marks; allow 1 mark for
incorrect joining of molecules that correctly shows removal of a molecule of water) [1]
ii) Condensation [1]

3 a) i) The top one (no marks); All C–C bonds single/no C=C double bonds [1]
ii) Condensation [1]
(Accept esterification)
b) Mix with alcohol; (decant into fresh tube and) add water and look for milky emulsion [2]
c) (Similarity) One of: Both have glycerol/fatty acids; (Difference) (Phospholipid has) phosphate
group/two fatty acids (not 3) [2]
(For similarity, allow both have ester bonds)

© Pauline Lowrie and Mark Smith 2015


1 Biological molecules Answers

4 a)

Statement Proteins Polysaccharides Lipids

Molecule is a polymer   x

Contains amino acids  x x

Contains nitrogen atoms  x x

(3 marks, 1 per correct row)


b) i) One [1]
ii) Any two of hydrogen bond; ionic bond; disulfide bond [2]

© Pauline Lowrie and Mark Smith 2015


2 Enzymes Answers

Test yourself on prior knowledge (page 19)


1 Two from: temperature, pH, concentration, surface area
2 The substrates cannot bind to the sites on the enzyme.

Test yourself (page 24)


1 The substrate is lactose, and the products are glucose and galactose.
2 The diagram should show an enzyme with an active site that is the right shape for starch to
fit into, but not cellulose, which is a different shape.
3 Maltose is not the same shape as it wouldn’t fit into the active site. The student should have
said that maltose is a complementary shape to the active site, or that maltose is a shape that
fits into the active site.
4 The enzyme lowers the activation energy needed for the reaction to take place, so high
temperatures and pressures are not needed.

Test yourself (page 27)


5 An enzyme is involved in the formation of uric acid. A molecule of allopurinol is similar in
shape to that of the normal substrate of this enzyme. Allopurinol blocks the active site, so
uric acid will not be formed.
6 Alcohol is similar in shape to ethylene glycol. It acts as a competitive inhibitor of alcohol
dehydrogenase. This reduces the rate at which ethylene glycol is broken down, so the toxic
metabolites are produced more slowly.
7 Adding more substrate ‘dilutes’ the competitor so that a substrate molecule has a greater
chance of entering the active site than the competitive inhibitor. However, non-competitive
inhibitors do not enter the active site, so adding more substrate will make no difference.
8

© Pauline Lowrie and Mark Smith 2015


2 Enzymes Answers

Required practical 1: Investigation into the effect of a named


variable on the rate of an enzyme-controlled reaction
(page 30)
1
pH Initial length of egg Final length of egg % egg white that
white in tube/mm white in tube/mm has been digested
4.8 54 47 13.0
5.6 50 45 10.0
5.8 52 42 19.2

6.2 54 40 25.9
6.6 52 32 38.5
6.8 53 27 49.0
7.2 52 17 67.3
7.6 48 7 85.4
8.2 47 15 68.0
8.6 52 23 55.8
9.0 54 37 31.5
9.6 53 49 7.5

2 Graph should have an approximately bell-shaped curve.


3 All the tubes had a slightly different length of egg white in them to start with, and this
allows us to compare the results.
4 This would be the same volume of buffer solution with either boiled enzyme or distilled
water. This shows that it is the enzyme digesting the egg white and not the buffer solution.
5 This prevents evaporation so that the concentration of enzyme remains constant.
6 Buffers maintain the pH even if small amounts of acid or alkali are added. If acid or alkali
was used to create the pH, the pH would vary over the period of incubation.
7 Of all the pH values tested this gave the greatest digestion of egg white. However, she would
need to repeat this to be sure, and to repeat using pH values around 7.6, i.e. 7.4, 7.5, 7.7, 7.8,
etc.

Test yourself (page 33)


9 Digestion involves enzymes. As the temperature increases so does the kinetic energy (of the
molecules in the gut). At higher temperatures there will be more collisions and a faster rate
of digestion/faster rate of action of these enzymes.

© Pauline Lowrie and Mark Smith 2015


2 Enzymes Answers

10 pH is a logarithmic scale/pH is not an arithmetic scale; so a small change in value


represents a large difference in the amount of hydrogen ions present and therefore pH
value.
11

12

13

14 This reduces the kinetic energy of the enzyme molecules, so hydrogen bonds are less likely
to break. This means the enzyme’s tertiary structure is stable and it is less likely to
denature.

Activity: Making a better oral rehydration solution (page 33)


The following points need to be considered in designing this investigation:
• The variable that you are changing is the concentration of amylase. Therefore all other
variables need to be kept the same.
• A standard rice-flour solution needs to be made up, using the same kind of rice flour and a
known concentration.
• Various different concentrations of amylase need to be made.

© Pauline Lowrie and Mark Smith 2015


2 Enzymes Answers

• Several different tubes/flasks need to be made up, each with the same volume or mass of
rice flour solution.
• These need to be placed in a water bath at a constant temperature, e.g. 37 °C.
• The same volume of amylase solution needs to be added to each tube/flask, but each tube
will have a different concentration of amylase added to it.
• A control should be done: this will be the same as the experimental tubes but using boiled
amylase solution or distilled water instead.
• After a fixed time period, e.g. 5 minutes, the viscosity of each tube must be measured. You
may have many different ways of testing this. One method might be to remove exactly 10
cm3 of mixture from each tube using a graduated pipette, hold the pipette above an empty
tube, and time how long it takes for the pipette to empty completely.
• Think of the results you will record and a graph you will plot. For example, you might plot a
graph with amylase concentration on the x-axis and time taken for pipette to empty in
seconds on the y-axis.

Practice questions (page 34)


1 a) i) A 1% solution contains 1 g of solute in 100 g (i.e. 100 cm3) water so you would need
0.2 g of starch dissolved in water and made up to 20 cm3 in volume. (Correct answer
2 marks. Allow 1 mark for figures that would give a 1% solution of a different volume,
e.g. 1 g starch in 100 cm3 water.)
ii) To keep pH constant. Temperature is the independent variable so other variables must
be kept constant. [2]
iii) To ensure that the contents had all reached the temperature of the water bath; ensures
reaction takes place at a constant temperature. [2]
b) 2 cm boiled enzyme OR 2 cm distilled water; to show it is the enzyme that digests the starch
3 3

and not any other factor. [2]


c) Mass/volume of starch digested/time. (2 marks. Allow 1 mark for time taken.)

2 a) Amino acid; it has the general structure of an amino acid; with a central carbon atom to
which is attached an amino group, a carboxylic acid group, hydrogen and an R-group [3]
b) i) Glucosamine 6-phosphate is a molecule with a very different shape to that of the
substrate/glutamic acid; therefore it will not fit into active site. [2]
ii) Binds to the enzyme/glutamine synthetase somewhere other than the active site; this
distorts the shape of the active site; the substrate/glutamic acid no longer fits [3]
c) i) Stops the pathway when enough product has been formed; excess wastes
energy/wastes (initial) substrate [2]

© Pauline Lowrie and Mark Smith 2015


2 Enzymes Answers

ii) Prevents excess intermediates from being formed; these could be poisonous/of no use
to organism [2]
iii) Will inhibit/control the reaction; even though not similar in shape/structure to
substrate [2]

3 a) To keep concentrations of gelatine constant; to keep concentration of pineapple extract


constant; tube B had HCl added [2]
(‘to keep concentration constant’ is acceptable for 1 mark if points 1 and 2 are not made)
b) Tube A: enzyme (in pineapple) has digested gelatine; tube B: enzyme denatured/inhibited; by
HCl/change of pH; reference to hydrogen bonds/tertiary structure changed [4]
c) For comparison/as a control; to show that it is an enzyme in pineapple that digested
gelatine/stopped gelatine setting in tube A; boiling denatures enzyme; other components of
pineapple still present [3]

© Pauline Lowrie and Mark Smith 2015


3 Cells Answers

Test yourself on prior knowledge (page 36)


1 Mitochondria release energy in respiration, so a lot of them are needed in cells in an insect
wing as these muscles use a lot of energy when the insect is flying. Cells in the gut do not
need as much energy so they have fewer mitochondria.
2 Animal cells do not have a cell wall, chloroplasts or a large permanent vacuole (any 2).
3 Enzymes are made of protein, and ribosomes make proteins. Therefore, lots of ribosomes
are needed to produce enzymes.
4 It forms long straight chains that hydrogen bond together to form strong fibres. It is also
insoluble.

Test yourself (page 39)


1 a) 1 000
b) 1 000 000
2 Measured length = 41mm = 41000 μm
Actual length = measured length/magnification
= 41000/3300 μm
= 12.4 μm
3 The membrane cannot be seen but it can be inferred that there is a membrane as the cells
have a definite ‘edge’.

Activity: Sections through different planes (page 39)


1

2 R.
3 The cell has been cut through at an oblique angle.
4 They are filled with haemoglobin and have very few cell organelles.
5 The answer will vary depending on the measurement made, but
magnification = measured size/real size.
If you measure the diameter of cell X as 50 mm, then magnification
= 50 000/7 = 7143 times.

© Pauline Lowrie and Mark Smith 2015


3 Cells Answers

6 The three-dimensional shape of the cells.

Test yourself (page 43)


4 Similar: (any three from) both have nuclei, Golgi bodies, mitochondria, rough and smooth
endoplasmic reticulum, cytoplasm, lysosomes, cell-surface membranes, ribosomes.
Different: plant cell has a cell wall, animal cell doesn’t; plant cell has a large vacuole; plant
cell contains chloroplasts.
5 Measure the length in millimetres, convert to micrometres by multiplying by 1000, then
divide by the magnification which is 16 000. For example, if you measure the length of the
chloroplast to be 96 mm, the actual size is 96 000/16 000 = 6 μm.
6 The rising glucose concentration inside the chloroplast would make the water potential
inside the chloroplast lower than the water potential outside the chloroplast. Water would
move by osmosis down the water potential gradient into the chloroplast. If enough water
entered, this would burst the chloroplast.

Test yourself (page 45)


7 Large surface area, short diffusion pathway, large concentration gradient
8 a) Optical microscope
b) Scanning electron microscope
c) Transmission electron microscope
9 This is because they are lipid soluble so they can diffuse through the phospholipid bilayer.
10 To keep the pH constant so that enzymes in the organelles are not denatured.

Required practical 4: Investigation into the effect of a named


variable on the permeability of cell-surface membranes
(page 47)
1 To allow any pigment from the cells that were damaged when the discs were cut to diffuse
into the water. This means that any pigment released in the subsequent investigation would
have come from intact cells.
2 The discs were handled carefully to make sure that no cells were damaged during handling,
as damaged cells might leak pigment into the solution. It is important that any pigment that
is released results from the effect of temperature on the beetroot membrane and not any
other factor.
3 To make sure the water in the tubes had reached the temperature of the water bath.
4 The beetroot pigment is pinky-red, so a blue-green filter is complementary.

© Pauline Lowrie and Mark Smith 2015


3 Cells Answers

5 This is a ‘blank’. It is used to set absorbance at zero for distilled water. This means that the
colorimeter will only record the absorbance changes that result from the presence of
beetroot pigment but not absorbance that results from the cuvette or the distilled water.
6 The graph shows little pigment leaking out of the membrane until around 40 °C. After this
pigment leaks out of the cells more rapidly. The higher the temperature after this, the more
pigment leaks out. There is a levelling-off after about 85 °C.
7 The membrane remains fairly stable until about 40 °C. At this point, the phospholipids and
proteins in the membrane have more kinetic energy so the pigment may be able to ‘leak’
through the phospholipid bilayer. In addition, the increased temperature causes changes in
the tertiary structure of the membrane proteins, making them more permeable to the
pigment. As the temperature increases above this, the phospholipid bilayer becomes more
disrupted, and the proteins more denatured, so even more pigment is able to leak out.
8 There are various answers here and only some can be mentioned. Cuvettes may differ very
slightly in thickness. A slightly thicker, or scratched, cuvette will absorb slightly more light
than a thinner unscratched cuvette. This can be overcome by using the same cuvette for
every reading, or repeating the investigation many times and finding a mean. The beetroot
discs are not completely identical in size and shape so some tubes could have slightly more
beetroot tissue in than others. This is difficult to overcome except by cutting the discs as
accurately as possible using a scalpel and ruler, and by repeating each investigation several
times to find a mean. Some parts of the beetroot tissue have more pigment in their cells
than others; again this can be overcome by several repeats, using different parts of the
beetroot and finding a mean that is representative of the beetroot as a whole. The
temperature of the water baths may not be exactly uniform; this can be overcome by more
careful monitoring or using thermostatically controlled water baths.

Required practical 3: Production of a dilution series of a solute


to produce a calibration curve with which to identify the water
potential of plant tissue (page 49)
1
Dish Volume of 1.0 M sodium Volume of
chloride solution/cm3 distilled
water/cm3
1 1.0 M sodium chloride 20 0

2 0.75 M sodium chloride 15 5

3 0.5 M sodium chloride 10 10

© Pauline Lowrie and Mark Smith 2015


3 Cells Answers

4 0.25 M sodium chloride 5 15

5 0.0 M sodium chloride 0 20

2 Calculate the percentage change in mass using the formula:


initial mass −final mass
change in mass (%) = × 100
initial mass

You should have a positive figure for a gain in mass, and a negative figure for a loss in mass.

Dish Initial Final Percentage


mass/g mass/g change in mass
1 1.0 M sodium chloride 3.83 2.87 −25.1

2 0.75 M sodium 4.07 3.52 −13.7


chloride
3 0.5 M sodium chloride 4.02 3.56 −11.4

4 0.25 M sodium 3.99 3.84 −3.8


chloride
5 0.0 M sodium chloride 3.96 4.73 +19.4

3 If the tissue is damaged water may be squeezed out of cells reducing the final mass. The
discs must be blotted dry carefully, as any excess solution on the discs will increase the
mass of the discs.
4 No, this does not matter. They were similar in size and shape, and by calculating the
percentage change in mass the results for each dish can be compared.
5 The graph should cross the x-axis at a point between 0.0 and 0.25 M sodium chloride. The
value where the line crosses the x-axis is the molarity of sodium chloride that has the same
water potential as the potato tissue.
6 There is no one correct answer here, but you should understand that 0.25 M sodium
chloride and more concentrated solutions are hypertonic to the potato cells. This means
they have a lower water potential than the potato cells so the potato cells lose water by
osmosis. 0.0 M sodium chloride (i.e. distilled water) is hypotonic to the potato cells so it has
a higher water potential than the potato cells. Water enters the potato cells by osmosis. The
sodium chloride solution where the line on the graph crosses the x-axis is isotonic with the
potato cells.

© Pauline Lowrie and Mark Smith 2015


3 Cells Answers

7 The possibility of not blotting the discs dry correctly has already been mentioned so other
limitations should be found. One limitation is the small errors in using a graduated pipette,
so the molarities of the sodium chloride solutions may not be completely accurate. This can
be overcome by using an automatic fixed volume pipette. Another limitation is that the
potato tissue may not be completely uniform so discs from one part of the potato may have
a slightly different water potential from cells in another part of the potato. This can be
overcome by repeating the investigation several times, using discs from different parts of
the potato. The mean should then be representative of the tissue as a whole. Other
limitations may be evaporation of solution from the dishes during the investigation; errors
in measuring the mass of tissue, which would be more accurate if a balance showing three
decimal places was used; and only five different molarities of sodium chloride solution
being used.

Test yourself (page 50)


11 Facilitated diffusion uses protein carriers while simple diffusion is diffusion through the
phospholipid bilayer.
12 Similarity: both use protein carrier molecules. Differences: active transport uses energy
from respiration while facilitated diffusion does not; active transport transports molecules
against a concentration gradient while facilitated diffusion transports substances down a
concentration gradient.
13 Proteins have complex three-dimensional shapes, so only the molecule that the protein
transports will fit into the protein.
14 Similarities: both processes involve movement of substances down a concentration
gradient and do not use additional energy. Differences: osmosis involves water only, and a
membrane must be present.
15 This is so that the blood cells do not take in water or lose water by osmosis.
16 a) So that the organelles, e.g. mitochondria, do not take in water by osmosis and burst.
b) So that the enzymes present are inactive.

Test yourself (page 52)


17 Similarities: both have cell membranes, ribosomes, DNA.
Differences: prokaryotic cells do not have a nucleus or membrane-bound organelles such as
mitochondria; prokaryotic DNA is circular and not complexed with proteins, while
eukaryotic DNA is linear and associated with proteins; prokaryotic cell walls are made of
murein, while eukaryotic cell walls, if present, are not; prokaryotic ribosomes are smaller
than eukaryotic ribosomes; prokaryotic cells sometimes have a capsule around them but
eukaryotic cells do not.
18 Magnification = measured length/real length = 40 000/2 = 20 000

© Pauline Lowrie and Mark Smith 2015


3 Cells Answers

Practice questions (page 53)


1 a) A = cell wall; B = flagellum; C = plasmid [3]
b) [3]

Name of structure Description

Rough endoplasmic Synthesises proteins and transports them around


reticulum the cell

Lysosome Contains enzymes that digest substances, such as


worn-out organelles or ingested bacterial cells

Microvilli Increases the surface area of the cell.

2 a) [2]

Uses membrane proteins Requires energy from ATP

Active transport  

Facilitated diffusion  X

b) Water potential in hollow/in sucrose solution lower/more negative than potato cells; water
enters solution by osmosis; down a concentration/diffusion/water potential gradient. (For
first marking point, answer must refer to water potential and not concentration.) [3]

3 a) The water potential of the sodium chloride solution was lower/more negative than the water
potential of the potato cells/tissue; water leaves potato cells; by osmosis; cells smaller/less
turgid/loss of mass; (also acceptable: reverse argument for marking point 1). [3]
b) Allows comparison; discs differ in mass to start with. [2]
c) 0.27 M (acceptable: 0.26–0.28 M) [1]
d) i) So that the changes in mass were greater; reduces effect of measurement errors;
increases surface area so equilibrium reached sooner. [2]
ii) Surface area of discs similar/same; same distance to centre of discs (for water
movement). [2]

4 a) Fluid = molecules can move around/it is liquid; mosaic = proteins floating among
phospholipids/not just phospholipids/other molecules in it. [2]
b) i) Rate of entry into cell levels off at a certain point; this is when all the protein carriers are
occupied. [2]

© Pauline Lowrie and Mark Smith 2015


3 Cells Answers

ii) [2]

Molecule Description

A Small, lipid-soluble

B Water-soluble ion

Can pass through phospholipid bilayer; enters by simple diffusion/does not need protein
carrier/channel. (No marks for identifying A or B.)
c) i) Increased temperature causes pigment to leak from cells; slowly at first/up to 42°C but
greater after this/suitable named point. Also acceptable as alternative to second point:
no leakage of pigment up to 22°C but increased leakage above this temperature. [2]
ii) Heat denatures membrane proteins/described; allows pigment to leak; correct
reference to heat disrupting phospholipids. [2]

© Pauline Lowrie and Mark Smith 2015


4 DNA and protein synthesis Answers

Test yourself on prior knowledge (page 56)


1 Gene, chromosome, nucleus
2

3 It allows more space for haemoglobin.


4 When red blood cells are formed by mitosis, they have a nucleus to control their
development until they are mature.

Test yourself (page 60)


1 8
2 The particular organic base it contains
3 There are hydrogen bonds between each pair of organic bases. Many hydrogen bonds
acting together are collectively strong.
4 A condensation reaction occurs when two molecules are joined together and form water in
the process. The condensation bonds between nucleotides are called phosphodiester bonds.
5 The coiling helps to keep the two strands together, contributing to the stability of the
molecule.
6 If there is 28% cytosine, there must also be 28% guanine. These two bases make up 56% of
the total. The remainder is 44% so there must be 22% adenine and 22% thymine.

Test yourself (page 63)


7 The DNA of prokaryotic cells is short, circular and lacks non-coding regions or introns.
8 Because the experiment showed that only the bacteriophage DNA entered the E. coli and
ended up in new bacteriophages. If histone protein had been associated with this DNA, it
would also have entered the bacteria and both would have been passed on to new
bacteriophages. It would not have been possible to say for certain that DNA was the genetic
material.
9 a) Exons are coding regions of DNA and introns are non-coding regions within genes.
b) A gene is a base sequence of DNA that codes for the amino acid sequence of either a
polypeptide or a functional RNA whereas a locus is the position of a gene on a
chromosome.
10 The two types of non-coding DNA in eukaryotes are introns within genes and multiple
repeats of base sequences between genes.

© Pauline Lowrie and Mark Smith Limited 2015


4 DNA and protein synthesis Answers

Activity: Types of nucleic acid (page 68)


1
Type of Hydrogen bonds Number of Anticodon
nucleic acid present or not polynucleotide present?
present? strands in molecule
DNA  2 X
mRNA X 1 X

tRNA  1 

2 ACUACUACC

Test yourself (page 69)


11 A DNA triplet is a set of three bases in a DNA sequence that codes for an amino acid. An
mRNA codon is a set of three bases in a mRNA sequence that codes for a specific amino acid.
12 Because the DNA of prokaryotes lacks introns; there are no introns in transcribed mRNA to
be removed.
13 Ribosomes move along a molecule of mRNA in a 5′ to 3′ direction.
14 Exons can be spliced together in different ways to form different mRNA molecules. Different
ways of splicing the exons in the same pre-RNA means that more than one polypeptide
chain can be made from a single eukaryotic gene.
15 The genome is the full set of genes contained within a cell, whereas the proteome is the full
range of proteins made by a cell.

Practice questions (page 72)


1 a) i) X = deoxyribose, Y = adenine, Z = phosphate group [3]
ii)

[1]
b) Covalent bonds [1]
c) Phosphodiester bonds [1]
d) It is long, so it can store a lot of information. It is coiled so the information store is compact. [4]

2 a) 6 [1]

© Pauline Lowrie and Mark Smith Limited 2015


4 DNA and protein synthesis Answers

b) CAC [1]
c) CAC [2]
d) Because the gene may include introns and some triplets may be start or stop sequences. [2]
e) i) middle 20%, end 26% [2]
ii) Because the two regions of the chromosome have different genes which code for
different proteins and so have different DNA base sequences. [2]

© Pauline Lowrie and Mark Smith Limited 2015


5 The cell cycle Answers

Test yourself on prior knowledge (page 73)


1 i) egg 7
ii) zygote 14
2 The components of a DNA nucleotide are deoxyribose, a phosphate group and one of the
organic bases adenine, cytosine, guanine or thymine.
3 The four organic bases are adenine, thymine, guanine and cytosine. Adenine is
complementary to thymine and cytosine is complementary to guanine.

Test yourself (page 76)


1 TAGCTGC
2 Condensation reaction
3 A strand of DNA used as a pattern for the assembly of complementary bases into a new
strand of DNA.
4 DNA helicase
5 DNA polymerase can only form phosphodiester bonds at the 3′ end of a new strand. As a
replication fork opens, one new strand can be built in this direction. This is called
continuous replication. The 3′ end of the other new strand points the other way. This means
the DNA polymerase can only build the new strand in a series of short sections as the
template strand is exposed. This is called discontinuous replication.

Activity: Experimental evidence for semi-conservative


replication of DNA (page 77)
1 They switched the N isotope available to bacteria for making new nucleotides. This was so
that any further new strands of DNA would be made using nucleotides containing the 14N
isotope. This allowed them to tell the difference between original strands and new strands.
2 After just one generation, cell division and therefore DNA replication would have occurred
just once. They wanted to see what the composition of the DNA was after replicating just
once.
3 Since the band in the tube is in between the positions in the other two tubes, the density of
the DNA.

Test yourself (page 79)


6 A clone is a group of genetically identical cells.
7 A chromosome is a tightly coiled molecule of DNA. Chromatids are replicate chromosomes
joined at their centromere immediately following DNA replication.
8 They would appear as single structures rather than double structures.

© Pauline Lowrie and Mark Smith Limited 2015


5 The cell cycle Answers

9 Centromeres separate into two at anaphase.


10 Cytokinesis in plant cells involves vesicles fusing to extend the cell membrane across the
cytoplasm followed by new cell walls forming. In animal cells, the cell membrane is pulled
inwards across the centre of the cell, separating the cytoplasm into two.

Required practical 2: Preparation of stained squashes of cells


from plant root tips; set-up and use of an optical microscope
to identify the stages of mitosis in these stained squashes and
calculation of a mitotic index (page 80)
1 Early prophase
2 Anaphase
3 Telophase. Vesicles are fusing to form new cell membranes across the cytoplasm to
separate the cell into two daughter cells.
4 Metaphase
5 A, C, F, B, E, D
6 Telophase, because there are two nuclei visible in the cell.
7 0.26

Test yourself (page 82)


11 Growth and repair of tissues
12 One at the start of G1 and two at the start of G2
13 Any two routine metabolic processes. Examples include protein synthesis and active
transport.
14 In binary fission, both the circular DNA molecule and any plasmids in the cell undergo DNA
replication. Unlike mitosis, there is no nuclear envelope to break down and there are no
spindle fibres.

Test yourself (page 83)


15 Tumours form as a result of uncontrolled cell division.
16 Methotrexate stops cells replicating DNA because it prevents them from making new DNA
nucleotides.
17 Drugs will inhibit any mitosis, not just the uncontrolled mitosis in tumour cells.

Practice questions (pages 84–6)


1 a) B, D, A, C [1]
b) C, B, E, A, D [1]
c) i) 0.37 [1]

© Pauline Lowrie and Mark Smith Limited 2015


5 The cell cycle Answers

ii) 666 minutes [2]


iii) 96 minutes [2]

2 a) i) After 10 minutes, because movement stopped. [2]


ii) Anaphase starts at 100 minutes, at a distance of 20 μm. After a further 100 minutes,
the distance has decreased to 4 μm. So the mean rate of movement is 16/100 which is
0.16 μm min–1 [1]
iii) Extend the curve beyond 200 minutes until it intercepts with the axis. It intercepts at
250 minutes. A tangent drawn to the curve between 200 and 250 minutes back to the
distance axis gives a distance moved (AB) of between 12 and 14 μm. Divided by a total
time of 250 minutes (BC) the rate of movement is AB/BC which is between 0.05 and
0.06 μm min-1 [2]
b) i) Vincristine would inhibit anaphase, because the spindle fibres are used to move the
chromatids to the poles of the cell. [2]
ii) Hair growth and blood-cell replacement both depend on cell division. Treatment with
vincristine prevents mitosis in tissues other than the tumour being treated. [2]

3 a) DNA replication [1]


b) S phase [1]
c) Samples 3 and 4 [1]
d) The daughter cells resulting from budding are smaller. Binary fission gives two equally sized
cells. [1]
e) In mitosis, the nuclear envelope breaks down and spindle fibres form. There are no plasmids
to replicate. In binary fission, both the circular DNA molecule and any plasmids in the cell
undergo DNA replication. Unlike mitosis, there is no nuclear envelope to break down and
there are no spindle fibres. [3]

4 a) i) Your diagram should be the same as the third tube but with the top band thicker. [2]
ii) After one division of the bacteria on 14N medium, their DNA is a mixture of 14N and 15N
because the band of extracted DNA settles at a higher point in the tube since it is less
dense. However, it is not as high as DNA that contains just 14N. Its density is
intermediate between all 14N DNA and all 15N DNA which suggests that after just one
division, half the DNA still contains 15N.
b) i) Two bands, top and bottom positions, same thickness [3]
ii) Two bands, top and bottom positions, both thicker [2]

© Pauline Lowrie and Mark Smith Limited 2015


6 The immune system Answers

Test yourself on prior knowledge (page 87)


1 By damaging/destroying cells and producing toxins
2 This is because each antibody will bind only to one antigen.
3 By engulfing pathogens/phagocytosis
4 This is because the vaccine has caused the body to produce antibodies against the measles
virus. Therefore, if you are exposed to the virus, antibodies will already be present.
5 The cold virus keeps changing its antigens, so antibodies against one cold virus won’t fit
with the antigens of a different cold virus.

Test yourself (page 90)


1 Pathogenic means disease-causing; a pathogenic organism is a disease-causing organism.
2 You know from GCSE that competition for resources exists between different populations
that live together. Competition from non-pathogenic bacteria could prevent the population
growth of pathogenic bacteria. Some bacteria produce toxins. Toxins released by one
population of gut bacteria could prevent the growth of other populations.
3 The monomers from which bacterial carbohydrates, lipids and proteins are made are the
same as the ones that our own bodies contain. In addition, monomers do not cause an
immune response.
4 Blood transports heat released by active tissues. If more blood, and more tissue fluid, is
present at a site of infection, more heat will also be transported there.

Test yourself (page 93)


5 Enzymes have an active site but antibodies have a binding site, or receptor site. Enzymes
speed up the rate of a chemical reaction whereas antibodies bind to antigens.
6 Plasma cells manufacture, and release, antibodies made of protein. Mitochondria produce
ATP, which provides the energy for the condensation of amino acids to form a protein.
Rough endoplasmic reticulum contains ribosomes, which produce proteins by the
condensation of amino acids.
7 Only a small number of the particular B cells remain after infection. Many more memory
cells remain, making a collision more likely.
8 A TH cell will only react to an antigen if it is on the surface of an antigen-presenting cell
whereas a B cell reacts to free antigen. A TH cell responds by stimulating other immune
reaction, by, e.g., releasing chemical messengers, such as cytokines, whereas a B cell
responds by releasing its surface receptors as antibodies.
9 The cells of a single strain of bacterium carry many surface proteins, which act as antigens.
By chance, some of the B cells a human randomly produces will carry surface receptors

© Pauline Lowrie and Mark Smith 2015


6 The immune system Answers

specific to each of the antigens on the bacterial cells. Each sensitised B cell will respond by
rapid cell division to produce a clone of cells releasing its one specific antibody.

Test yourself (page 102)


10 Smallpox antigens do not change but other pathogens such as HIV have different strains
with different antigens. Also, the symptoms of smallpox are easy to recognise so people
with smallpox can be isolated from other people, but this is not so easy for other diseases.
For example, it is a long time before a person infected with HIV shows any symptoms, by
which time they could have passed the infection on to many other people. Also, HIV evades
detection because it hides inside cells.
11 Helper T cells release cytokines that stimulate other immune cells. So, with fewer helper T
cells, there will be fewer B cells maturing into plasma cells, therefore fewer antibodies;
fewer memory B cells formed; and fewer cytotoxic T cells will be activated.
12 These proteins are the right shape to fit into receptor proteins on the surface of helper T
cells.
13 Screening blood transfusions; use of condoms during sexual intercourse; use of sterile
needles for intravenous drug abusers, e.g. needle-exchange schemes; screening pregnant
women or anyone who may be at risk so that they know they are infected and can avoid
passing the infection on to others. Another suggestion might be education of the general
public so that they are aware of how the disease is spread.
14 This molecule will fit into the DNA molecule instead of the ‘real’ nucleotide. This means that
it cannot base-pair with the nucleotide in the complementary strand. Also it will introduce
an error into one triplet, so that triplet will not code for an amino acid and therefore a non-
functional protein will be made.

Test yourself (page 103)


15 So that any unbound antibody is washed away. If you didn’t wash it away, some of the
antibody with enzyme attached could stay in the well, and the enzyme could then change
the colour of the substrate in stage 6, giving a false positive result.
16 This is needed to detect the PSA antigen that has bound to the first antibody, and also to
introduce the enzyme.
17 If the blood did not contain PSA, no antigen would bind to the antibodies in the well.
Therefore, the second antibody would not bind in stage 4 and any unbound antibody would
be washed away. When the colourless substrate is added in stage 5, no enzyme would be
present to change its colour so the well would stay colourless.

© Pauline Lowrie and Mark Smith 2015


6 The immune system Answers

18

19 hCG is a specific shape that binds to the specific antibody. Other antigens are a different
shape and will not bind to this particular antibody.
20 This is so that there is a visible change on the dipstick, as the antibodies themselves are too
small to see.
21 The first row of antibodies detects hCG if it is present. The second row detects the anti-hCG
antibodies if they have not bound to hCG. It acts as a control to show the test is working,
and confirms that the woman is not pregnant.
22 The antibodies in the row labelled ‘5’ bind to hCG and therefore bind to any anti-hCG
antibodies that have bound to hCG. The second row of antibodies are antibodies that bind to
the unbound anti-hCG antibodies.

Activity: To vaccinate or not? A parent’s dilemma


(pages 104–5)
1 While a baby is breast-fed, it is protected against many infectious diseases by its mother’s
antibodies which are present in breast milk. When a baby is weaned, it is no longer
protected from infections in this way.
2 If boys have rubella, they can pass on the infection to pregnant women. By vaccinating
children of both sexes, it means that rubella is less prevalent in the population so a
pregnant woman is much less likely to be exposed to this virus.
3 The vaccine carries much less risk than being infected with measles.
4 This is a very small sample and therefore not reliable.

© Pauline Lowrie and Mark Smith 2015


6 The immune system Answers

5 No. First of all, the sample size is very small. Secondly, there was no control group. It is
possible that most children, whether or not they have autism, have traces of the measles
virus in their guts.
6 This means that Dr Wakefield had a conflict of interest. He was being paid by the parents to
show that MMR causes autism, and therefore could not be unbiased.
7 No. The upper graph shows that the percentage of children receiving the MMR vaccine by
24 months only increased very slightly between 1980 and 1994. The percentage of children
vaccinated by age 17 months did increase a little over this period of time. However, the
number of children diagnosed with autism increased by about 600% over the same time
period, which is a much greater increase. It may be that doctors became much better at
diagnosing autism over this period.
8 Because all children diagnosed with autism would have been vaccinated against MMR
recently.

Practice questions (pages 106–7)


1 a) Large molecule/protein/glycoprotein on outside of virus;
That causes an immune response/antibodies to be made; [2]
b) i) Takes time for antigen presentation/B cells to be activated/plasma cells to divide/B cells
to produce antibodies. [2]
ii) Memory cells were present; produce antibodies immediately [2]

2 a) i) P = glycoprotein on membrane/lipid envelope/phospholipid bilayer;


Q = capsid/capsomere [2]
ii) Carries genetic information [1]
b) Inserts genetic material into human cell/RNA; DNA copy made of viral RNA; inserted into host
DNA/chromosomes; cell makes new viruses; viruses released from cell (max 3).

3 a) i) [3]

Primary response Secondary response

Response 1 

Response 2 

Response 3 

ii) Response 1: no memory cells present; Response 2: memory cells present. Response 1:
the few B cells present must multiply/produce plasma cells and then produce the

© Pauline Lowrie and Mark Smith 2015


6 The immune system Answers

antibodies; Response 2: memory cells produce large numbers of plasma cells more
quickly. [3]
b) i) Antigen A gives secondary response/response 2; memory cells already present against
antigen A; Antigen B gives primary response/response 3. [3]
ii) To show that the response to one antigen is independent of the response to another
antigen. [1]

4 a) i) Cancer cells have specific (protein) receptors on cell surface; not present on other cells. [2]
ii) Radioactivity targeted at cancer cells; less likely to harm healthy cells; lower dose of
radioactivity needed (max 2).
iii) Antibody is protein; would be digested in gut. [2]
b) Antibody is different (shape) from self proteins; recognised as foreign; human mounts
immune response/makes antibodies against it. [3]

5 a) Inject (small amounts of) venom several times; to produce secondary response/many
memory cells. [2]
b) Toxin acts as antigen; activates B cell; stimulated by helper T cells; divides to form clone of
plasma cells; produces specific antibodies (max 4).
c) Antibodies bind to antigen; so cannot cause harm/pain. [2]
d) Sheep antibodies are more similar in shape to human antibodies; less likely to be seen as
‘foreign’ by human; therefore less/no immune response occurs. [3]

© Pauline Lowrie and Mark Smith 2015


7 Gas exchange Answers

Test yourself on prior knowledge (page 108)


1 All cells of the body, in the mitochondria
2 For example, synthesis of proteins for growth or enzymes, muscle contraction, active
transport
3 glucose + oxygen → carbon dioxide + water + energy
4 The air breathed into the lungs has less oxygen so the diffusion gradient into the blood is
less and oxygen passes in more slowly. Therefore, the rate at which it reaches the muscles is
also lower.
5 Carbon dioxide diffuses into the blood because the concentration becomes higher in the
muscles. It is circulated by the blood to the lungs. Here it diffuses through the lung
membranes into the air spaces where the concentration is low. It is then exhaled.
6 If the concentration of carbon dioxide is high in the air that is breathed in, the concentration
in the air spaces in the lungs will rise. This will reduce the concentration gradient between
the blood and the air spaces so carbon dioxide will not diffuse out, or will diffuse out more
slowly. This will cause the blood to become more acidic.

Test yourself (page 113)


1 a)

Length of side Surface area Volume of Surface area :


of cube (cm) of cube (cm2) cube (cm3) volume ratio
1 6 1 6:1
2 24 8 3:1
3 54 27 2:1

b) As the cube gets larger, the surface area to volume ratio gets lower.
2 Insects have a fairly large surface area to volume ratio and the inside of their body has a
high-water potential. The air usually has a much lower water potential, so water would
evaporate from the insect across its body surface if it was not waterproof.
3 These rings make sure the tracheae do not close so that there is a constant supply of air.
4 It only opens to the air where there are spiracles, reducing evaporation. Spiracles may
close, and some are protected by hairs, both of which further reduce water loss.
5 Lactate reduces the water potential of muscle cells. This causes water to move from the
ends of the tracheoles into the muscle cells. This increases the surface area of the tracheoles
that is in contact with air.

© Pauline Lowrie and Mark Smith 2015


7 Gas exchange Answers

Test yourself (page 115)


6
Large Short diffusion Large concentration gradient
surface area distance
Many Surface of Capillaries remove oxygenated blood and bring
filaments lamellae made of fresh deoxygenated blood
each with a single layer of Ventilation brings fresh oxygenated water across
many thin, flattened the filaments continuously and removes
lamellae cells deoxygenated water
Counter-current exchange system

7 If a fish is out of water, the thin gill filaments stick together, reducing the surface area for
gas exchange.

Test yourself (page 117)


8 Shortest distance on diagram = 5 mm = 5000 μm;
Actual shortest distance = 5000/10000 = 0.5 μm
9 The tar forms a layer on top of the alveolar cells, increasing the diffusion distance.
10 This ensures that the whole of the red blood cell surface is exposed so gas exchange can
occur across the whole surface. When the cell is in contact with the capillary endothelium
the diffusion distance is reduced.

Activity: Analysing a photomicrograph (page 120)


1 They are cut through different sections, so some are cut through the middle, at the widest
point and others through a narrower section.
2 The formula is measured size/magnification (find the magnification using the scale bar).
3 From the scale bar we know 10 mm (10 000 μm) represents 100 μm, therefore:
magnification = 10 000/100 = ×100.
The actual size of the alveolus depends on which measurement you make, as it is not exactly
circular. The measured length of it at its widest part is 31 mm.
actual size = measured size/magnification = 31 000/100 = 310 μm

Test yourself (page 122)


11 a) 0.6 dm3 e) 666%
b) 12 f) 5 dm3
c) 7.2 dm3/min g) 1.2 dm3
d) 4.6 dm3

© Pauline Lowrie and Mark Smith 2015


7 Gas exchange Answers

Activity: Interpreting pressure changes during breathing


(page 123)
1 a) 0.45 dm3
b) At rest, because total lung volume in a man is about 5 dm3 and the increase in volume
was much less than 1 dm3.
2 The volume of air breathed in increases steadily and then levels off.
3 The pressure in the alveoli decreases to about −0.14 kPa then increases again to 0 kPa.
4 The contraction of the diaphragm and the external intercostal muscles which increases the
volume of the thorax.
5 Because air has been taken into the lungs and the pressure inside the lungs now equals the
pressure outside the lungs.
6 The volume decreases rapidly then at a slightly slower rate to 0.
7 The pressure increases in the lungs at first, to about 0.14 kPa, then decreases to 0 kPa again.
8 The pressure increases at first because the diaphragm relaxing and the intercostal muscles
relaxing reduce the volume of the thorax and increase the pressure. This causes air to be
expelled from the lungs, decreasing the lung volume. Once a lot of air has left the lungs, this
reduces the pressure in the lungs so the rest of the air in the lungs leaves slightly more
slowly than at the start of expiration.
9 This is because the breathing movements cause air to enter or leave the space between the
chest wall and the lungs, and the volume of the lungs is not altered.

Test yourself (page 123)


12 Alveoli, bronchioles, bronchi, trachea, mouth/nose
13 Sticky mucus in the sinuses; cilia that move mucus and trapped bacteria to the throat.
14 Thin walls reduce diffusion distance; large numbers increase surface area; large network of
blood capillaries into which the oxygen can diffuse.
15 Breathing movements replace air in the alveoli that contains with oxygen at a higher
concentration at frequent intervals; blood circulation moves red blood cells through
capillaries rapidly so there is a continuous supply of cells with low oxygen concentration.
16 Water leaks into the alveoli from the blood and evaporates into the air that is breathed out.
17 Diaphragm is flattened as muscles round its edge contract; rib cage is pulled upwards and
outwards as the external intercostal muscles contract.
18 The volume of air drawn into the lungs per breath when at rest.
19 The depth and the rate of breathing increase.

© Pauline Lowrie and Mark Smith 2015


7 Gas exchange Answers

Activity: Analysing the evidence of the study by Doll and Hill


(page 129)
1 You would expect a curve showing increasing deaths from lung cancer as smoking in
women increases, but with a time delay as cancer takes several years to develop.
2 The graph shows deaths from lung cancer increasing from 1916 to 1959 which correlates
with the increase in cigarette smoking. Cancers other than lung cancer are falling over this
time, which suggests that lung cancer in particular correlates with smoking.
3 a)

Percentage surviving
Age (years) Smokers Non-smokers
60 81 91
70 58 81
80 26 59

b)

Difference in
Age (years) percentage surviving
60 10
70 23
80 33

c) 10 years

Activity: Counting stomata (page 132)


1
Underside of leaf Top surface of leaf
Number of Number of Number of Number of
stomata in stomata per stomata in field stomata per
Leaf number field of view mm2 of view mm2
1 41 270 0 0
2 55 362 0 0
3 72 474 0 0
4 52 342 0 0
5 40 263 0 0
6 47 309 0 0
7 38 259 0 0
8 59 388 0 0
9 40 263 0 0
10 42 276 0 0
Mean 48.6 320.6 0 0
SD 10.92 70.99 n/a n/a

© Pauline Lowrie and Mark Smith 2015


7 Gas exchange Answers

2 This means stomata are unlikely to be blocked by drops of rain which would reduce gas
exchange and the stomata are less likely to be in direct sunlight, reducing water
evaporation.
3 The student would estimate the diameter of the field of view. Ideally this will be done by
using an eyepiece graticule and a stage micrometer (see page 506). An alternative (less
accurate) method is to view a millimetre ruler under the microscope and estimate the
diameter of the field of view by estimating using the divisions on the ruler. The area of the
field of view is calculated using the formula πr2 where r is the radius of the field of view in
millimetres. The number of stomata in the field of view is multiplied by 1/area to give the
number of stomata per square millimetre.
4 This enables the student to compare the stomatal density between leaves of different sizes
and shapes.
5 The plan needs to address how the plants will be treated. One method is to find an area with
high carbon dioxide concentration, e.g. a motorway verge, and compare the stomatal
density using plants of the same species growing here and in an area of normal carbon
dioxide concentration. In this case there are confounding variables such as rainfall, soil
quality and other air pollutants, for example. The mean carbon dioxide concentration
should be measured at both sites. An alternative method is to grow plants of the same kind
in two (or more) different glasshouses, keeping other variables constant, e.g. soil quality,
water availability and light intensity. The same number of leaves from each site should be
sampled using the method above. You should suggest repeating this method using several
different species of plant.

Test yourself (page 135)


20 Sand dunes hold little water so the plant is often in dry conditions. Dunes also get exposed
to salty water from sea spray. Conditions are very windy which increases water loss from
leaves.
21 It is cooler at night, so water loss is reduced by having the stomata open at night and closed
by day.
22
Large Short Large concentration gradient
surface area diffusion
distance
Almost the Gases only In sunlight, the mesophyll cells use up carbon dioxide
whole of the have to diffuse in photosynthesis all the time so the concentration of
surfaces of across a cell carbon dioxide inside the cells is always lower than in
the membrane to the air spaces outside. Similarly, oxygen is released in
mesophyll enter or leave photosynthesis so the concentration on oxygen inside
cells are in the cells the cell is always higher than in the air outside. (In
contact with darkness, only respiration occurs so the
air concentrations of these gases is reversed.)

© Pauline Lowrie and Mark Smith 2015


7 Gas exchange Answers

23 Large, thin and flat leaves have a larger surface area so there is a maximum area for
trapping sunlight for photosynthesis. This is also a large surface area over which water can
be lost, so it is an advantage to lose these leaves in winter when the available water may be
frozen.
24 Needle-like leaves have a reduced surface area to volume ratio which reduces water loss.
This means the tree can survive winter when the water in the soil may be frozen and
unavailable, without shedding its leaves.

Practice questions (pages 136–8)


1 Feature and explanation required for each mark. Examples include the following.
Rolled leaves: reduces water potential gradient/reduces air movement across stomata/traps
moist air/reduces surface area;
Sunken stomata: reduces water potential gradient/reduces air movement across stomata/traps
moist air/reduces surface area;
Thick cuticle: reduces transpiration via cuticle/increases diffusion distance/reduces evaporation;
Hairs: trap air which becomes damp/moist/humid;
Reduced leaves/spines: reduced surface area/fewer stomata. [3]

2 a) Prevents drying out/reduces evaporation; keep moist air inside/trap moist air [2]
b) Diffuses through spiracles; diffuses along tracheae and trachioles; dissolves in fluid at end of
tracheoles; diffuses into muscle cell (3 max)

3 a) i) <to be supplied once image is typeset> [2]


ii) By diffusion across cell surface membrane; (it is single-celled so) large surface area to
volume ratio. [2]
b) Water and blood flow in opposite directions; the concentration gradient is maintained all the
way along. [2]

4 a) For comparison; different fish have different body masses [2]


b) i) B swims more slowly than A; less oxygen used in respiration [2]
ii) C is air breathing; higher partial pressure/concentration of oxygen in air than in water [4]

5 a) i) Dissolves in fluid; diffuses into muscle cell. [2]


ii) Water/fluid enters (muscle) cell; by osmosis; air in tracheole diffuses directly into cell;
shortens diffusion pathway. [3]
b) i) Reduces the risk of drying out/conserves water. [1]

© Pauline Lowrie and Mark Smith 2015


7 Gas exchange Answers

ii) At a very low partial pressure of O2 spiracles open many times a minute but rate of
opening falls as partial pressure of O2 rises; rapid fall at first, then shallower (answer
must use suitable figures from the graph); more air is needed in tracheae when O2
concentration low; increases rate of flow of air through tracheal system. [3]

6 a) The rate of expiration was much lower before use; the volume expired per second was much
lower. [2]
b) i) Before use, 1 l; after use, 2.5 l [2]
ii) Draw a tangent to the graph; find the gradient. [2]
c) The man could breathe much more easily, taking in more air per breath and expiring more
rapidly. [2]
d) Less air had been inspired because of the airways being narrower, so there was less air to
expire. [1]
e) The drug in the inhaler made to muscles in the walls of the bronchi and bronchioles relax, so
the airways were wider and air could be drawn more easily and rapidly. [2]

7 a) About 70 m2 [2]
b) Suitable answer, for example not all the area is in contact with a capillary. Alveoli connect
with others so they are not complete spheres. [1]

© Pauline Lowrie and Mark Smith 2015


8 Digestion and absorption Answers

Test yourself on prior knowledge (page 140)


1 Muscle is made of many similar cells that perform a specific function. The stomach is made
of several different kinds of tissue, e.g. muscle, glandular and epithelial.
2
Name of enzyme Where produced Reaction Part of the gut
catalysed where reaction
occurs
Amylase Salivary glands Starch to maltose Mouth
Pancreas Small intestine
Protease Stomach Proteins to amino Stomach
Pancreas acids Small intestine
Lipase Pancreas Lipids to fatty acids Small intestine
and glycerol

3 This neutralises the acid contents of the stomach so that the pancreatic enzymes are at a
suitable/optimum pH.
4 pH, temperature

Test yourself (page 143)


1 The mitochondria produce a lot of ATP which is needed to actively transport sodium ions
out of the cell. This allows sodium ions to diffuse in from the lumen, down their
concentration gradient, bringing glucose with it.
2 It is the wrong shape to fit into the active sites of amylase or any of the digestive enzymes
present. Cellulase is not produced in mammalian guts.
3

4 Only glucose is the right shape to fit into its receptor site.

Test yourself (page 148)


5 This breaks the large protein into ‘chunks’ so that there are more ‘ends’ for the
exopeptidases to work on.
6 This protects the stomach lining from being digested by the protein-digesting enzymes
there.

© Pauline Lowrie and Mark Smith 2015


8 Digestion and absorption Answers

8 Similarities: Both involve several steps whereby the polymer is digested into smaller units
consisting of two monomers; the final stage of digestion for both uses an enzyme in the
membrane of the gut epithelium cell. Differences: protein digestion occurs in the stomach
and small intestine, carbohydrate digestion only in the small intestine.

Activity: Chromatography of amino acids (page 149)


1 There are traces of amino acids on your skin, from sources such as sweat. These will show
up on the paper if the paper is handled directly.
2 Ink would dissolve in the solvent so you would not be able to see it later.
3 This would wash the amino acids out of the paper.
4 Amino acids are colourless and would not show up on the paper unless a locating agent was
used.
5 Mixture = glutamic acid, leucine and arginine;
A = methionine;
B = arginine;
C = tyrosine;
D = leucine;
E = alanine

Test yourself (page 151)


9 Lipids can diffuse through the phospholipid bilayer so a carrier protein is not needed.
10 It breaks down large fat droplets into smaller droplets with a larger surface area. Lipase can
only work on the surface of the droplets so this makes lipid digestion more efficient.
11 It only occurs in the small intestine, and only involves a single enzyme.
12 So that they can be transported in the blood which consists mainly of water.

Practice questions (pages 152–3)


1 a) i) Increases surface area; for absorption (of products of digestion/correct example); for
enzymes. [3]
ii) Provides ATP; for active transport; of sodium ions out of the cell. [3]

© Pauline Lowrie and Mark Smith 2015


8 Digestion and absorption Answers

b) i) Lipase; from pancreas; in small intestine; lipids/triglycerides to fatty acids and glycerol. [3]
ii) Small-chain fatty acids diffuse through cell into capillary; long chain fatty acids and
glycerol and monoglycerides (any 2); recombine to form triglycerides; packaged into
chylomicrons; with cholesterol and phospholipids. [3]

2 a) i) All the glycosidic bonds have been broken/the number of glycosidic bonds broken is the
same as the initial number. [1]
ii) 50; there is still a glycosidic bond between every two glucose units/only half of the
glycosidic bonds will have been broken. [2]
b) i) Thermostable means that the enzyme continues to work at high temperatures/the
enzyme is not readily denatured at high temperatures; (the flow chart shows that)
bacterial amylase converts starch to liquefied starch at 105°C. [2]
ii) Not all glycosidic bonds have been broken as (the flow chart shows that) D.E. is 99/not
100. [2]
c) Mainly starch; but (the flow chart shows that) some glycosidic bonds have been broken to give
D.E. value of 11; so smaller molecules/substances such as maltose/glucose are present. [3]
d) i) Denatures the enzyme; therefore substrate will no longer fit active site (and be broken
down). [2]
ii) Enzyme hydrolysis requires lower temperatures/more energy efficient/does not need
corrosion-resistant apparatus. [1]

3 a) Digested by lactase in membrane of (gut) epithelial cell; glucose and


galactose/monosaccharides pass into cytoplasm. [2]
b) i) So that blood glucose level was normal/so that nothing else affected blood glucose
concentration. [1]
ii) Person B (no mark): blood glucose concentration does not go up/stays unchanged;
showing no digestion of lactose has taken place. [2]
c) (Increase in blood glucose concentration divided by blood glucose concentration at
0 minutes) x 100% [1]

© Pauline Lowrie and Mark Smith 2015


9 Mass transport in animals Answers

Test yourself on prior knowledge (page 154)


1 Cells becoming specialised to carry out particular functions
2 Diffusion would be much too slow to transport substances around multicellular organisms
because the distances are much larger than in single-celled organisms. Multicellular
organisms therefore depend on mass transport systems.
3 3 127 320 000; 3.12732 × 109
4 Glucose, oxygen

Test yourself (page 157)


1 Renal arteries
2 Elastic fibres allow the artery to expand each time the heart beats. Another advantage is
that the fibres recoil between heartbeats. This smooths out the changes in pressure and
maintains a more constant blood flow.
3 Muscle cells can contract and control blood flow to particular organs.
4 Valves in veins ensure one-way flow towards the heart.

Test yourself (page 159)


5 If plasma proteins escaped from blood capillaries, they would lower the water potential of
the tissue fluid until water could no longer be reabsorbed into the blood by osmosis.
6 If a diet contains very little protein, blood contains fewer plasma proteins. This means that
the water potential of the plasma is not low enough to reabsorb water by osmosis. Tissue
fluid collects in the tissues, which is called oedema.
7 Blockage of the lymph vessels prevents excess tissue fluid from draining back to the blood
circulation. Tissue fluid collects in the tissues, which is called oedema.
8 High blood pressure forces more fluid out of capillaries. Hands and feet are at the extremity
of the lymphatic system and the excess tissue fluid is slower to return to the blood
circulation.

Test yourself (page 160)


9 Centrifuging a sample of blood would separate the cells from the plasma. The cells would
collect at the bottom of the tube in a pellet.
10 Red blood cells lack nuclei, allowing more space for haemoglobin. Their shape provides a
high surface area to volume ratio, ensuring that all the haemoglobin is relatively near the
surface, so the diffusion distance for oxygen is always relatively short.
11 Since red blood cells lack nuclei, they have no genetic instructions for the synthesis of
proteins.

© Pauline Lowrie and Mark Smith 2015


9 Mass transport in animals Answers

Test yourself (page 165)


12 Different tissues respire and use oxygen at different rates, resulting in different partial
pressures of oxygen.
13 Haemoglobin loads oxygen in the lungs, where the partial pressure of oxygen is high.
Haemoglobin unloads oxygen in respiring tissues where the partial pressure of oxygen is
much lower.
14 The advantage of the Bohr effect is that when a tissue is respiring more and producing more
carbon dioxide, oxyhaemoglobin will dissociate more completely, releasing more oxygen.
15 Mammals that live at high altitude have haemoglobin with a different oxyhaemoglobin
dissociation curve to that of low-altitude mammals. It means that their haemoglobin can
become saturated with oxygen even at the lower partial pressure found at higher altitudes.

Required practical 5: Heart dissection (pages 167–9)


1
A = aorta G = pulmonary vein
B = pulmonary artery H = coronary arteries
C = left atrium I = inferior vena cava
D = coronary arteries J = right atrium
E = left ventricle K = right ventricle
F = superior vena cava

2
Structure Function
Right atrium Receives deoxygenated blood from the body
Aorta Carries oxygenated blood round the body
Right ventricle Pumps deoxygenated blood to the lungs
Vena cava Takes deoxygenated blood from the body to the heart
Left atrium Receives oxygenated blood from the lungs
Left ventricle Pumps oxygenated blood from the heart
Pulmonary vein Carries oxygenated blood from the lungs back to the heart
Pulmonary artery Carries deoxygenated blood from the heart to the lungs
Coronary arteries Carry oxygenated blood to the heart muscle

3 They stop the valve opening backwards, and ensure the valve stays closed when the
ventricle contracts.
4 It has to pump blood all round the body, and not just to the lungs as the right ventricle does.
5 Pulmonary vein, left atrium, AV valve
6 Left ventricle, aortic valve, aorta
7 Vena cava, right atrium
8 Right ventricle to right atrium via AV valve

© Pauline Lowrie and Mark Smith 2015


9 Mass transport in animals Answers

Test yourself (page 170)


16 This is so that they can pump the blood with enough pressure to go all the way to the lungs
(right ventricle) or all the way round the body (left ventricle).
17 The left ventricle pumps blood all around the body, so more pressure is needed. The right
ventricle only pumps blood to the lungs so less pressure is needed.
18 They pump the blood into the ventricles, so less muscle is needed for this.
19 When the ventricles contract the tendons become taut, preventing the pressure from
pushing the flaps of the valve any further.
20 The pressure of the blood is lower when it leaves the lungs because it has been through a
system of capillaries. This allows the blood to be pumped again so it can pass round the
body at greater pressure.
21 This means that oxygenated blood from the left side of the heart mixes with deoxygenated
blood from the right side of the heart. Therefore, blood leaving the left ventricle will not be
fully oxygenated. This may cause the child to become tired more easily, and become
breathless when undertaking physical activity.

Activity: Calculating risk of CHD (page 177)


1 a) Older people have had more time to accumulate damage to their cardiovascular system
as a result of all these risk factors, and also as a result of the ageing process.
b) If you have high blood cholesterol when you are young, this means that atheroma is likely
to develop faster and therefore you are more likely to have the amount of atheroma
normally associated with an older person.
c) High HDL levels protect against CHD, so if you have high HDL concentration in your
blood, you have a reduced risk of CHD compared to an average person.
2 a) Point score is 11. This means he has an 8% increased risk of CHD over the next 10 years.
Advice: give up smoking; take more exercise; eat a diet lower in fat and salt.
b) Point score is 11 so he has only a 8% increased risk of developing CHD over the next 10
years. Advice: he can’t do anything about his age, so the advice is to ensure he is eating a
low-fat and low-salt diet and takes regular exercise.

Practice questions (pages 178–80)


1 a) It means that the lugworm haemoglobin can become saturated with oxygen even at the
lower partial pressures that exist in its tube when the tide goes out. [2]
b) Quarternary structure means that the molecule contains more than one polypeptide chain. [1]

2 a) i) When the partial pressure of oxygen falls to the value found in tissues, a large
proportion of the oxygen is unloaded. [1]

© Pauline Lowrie and Mark Smith 2015


9 Mass transport in animals Answers

ii) When the partial pressure of oxygen reaches that found in the lungs, the haemoglobin is
almost completely saturated. [1]
b) i) The curve should be further to the right. [2]
ii) The advantage is that when a tissue is respiring more and producing more carbon
dioxide, oxyhaemoglobin will dissociate more completely, releasing more oxygen. [2]

3 a) [1]
Valve Open or closed?
Atrioventricular valve Closed
Semi-lunar valve Open

b) i) 80 cm3 [2]
ii) Cardiac output = stroke volume × heart rate = 80 × 75 = 6000 cm 3
[2]
4 a) Smokers are less likely to survive without developing heart disease than non-smokers/those
who have given up; non-smokers more likely to survive without heart disease than
smokers/those who have given up; those who give up smoking have a better chance of
surviving without heart disease than those who continue smoking. [3]
b) (Yes) Those who give up smoking have a better chance of surviving without heart disease
than smokers; after 3 years their chances of surviving without heart disease are similar/only a
bit less than those who have never smoked.
(No) The risk of developing heart disease has not fallen by 50%/a large amount; the graph
shows lowest risk for those who have never smoked; data do not distinguish between
heavy/light smokers; don’t know how many years smokers have been smoking. (4 max)

5 a) i) Some people have more than one risk factor. [1]


ii) Valid suggestion, e.g. genetic factors, diabetes. (High fat/saturated fat diet are not
acceptable, as this overlaps with blood cholesterol. Also high salt in diet is not allowed as
this overlaps with blood pressure.) [1]
b) 40 mg; produces maximum/almost maximum reduction in blood pressure; higher dose may
have side effects; cost-effective. [3]

© Pauline Lowrie and Mark Smith 2015


10 Mass transport in plants Answers

Test yourself on prior knowledge (page 181)


1 Water and mineral ions
2 It is unlikely to be toxic to animals. It is transported around the plant, so it can reach
underground parts of the plant and kill them too.

Test yourself (page 185)


1 The cells contain no cytoplasm so they form empty tubes. Their cell walls are reinforced
with lignin. Small gaps called pits in the side walls allow water to move between vessels if
any get blocked. The end walls are broken down so the cells form continuous pipes.
2 The polarity of water molecules. Polarity results in the cohesive forces between water
molecules that enable a column of water to be pulled up xylem vessels.
3 Water evaporating from leaves has a cooling effect, reducing the risk of leaves overheating
and their enzymes being denatured. The stream of water also transports mineral ions up
through the plant from the roots.
4 Warm, dry and windy conditions would result in the most rapid rate of transpiration.

Activity: Investigating translocation in phloem (page 188)


1 Radioactive carbon dioxide is absorbed by leaves and used in photosynthesis to make
sucrose. Since the sucrose is then radioactive, its movement around the plant by
translocation can be followed or traced.
2 The water supply to the leaves needs to remain intact during the experiment otherwise the
plant will wilt.
3 To show where sucrose is translocated in an intact plant.
4 Phloem transport in the stem occurs both upwards and downwards.

Test yourself (page 189)


5 Transfer cells carry out a lot of active transport which requires ATP. They have numerous
mitochondria to make sufficient ATP.
6 In the leaves, transfer cells actively load sucrose into the phloem. As sucrose accumulates in
the sieve elements, the water potential is lowered, so water moves into the sieve tube by
osmosis from nearby xylem vessels, increasing the hydrostatic pressure. In the potatoes,
transfer cells actively unload sucrose from the phloem. As sucrose is removed the water
potential increases, so water moves into the potato tissue by osmosis reducing the
hydrostatic pressure. The pressure gradient between the leaves and the potatoes causes the
mass flow of phloem sap towards the potatoes.

© Pauline Lowrie and Mark Smith 2015


10 Mass transport in plants Answers

7 Translocation of sugars is prevented if the phloem all around a tree trunk is damaged,
stopping transport of phloem sap up from the roots to new buds in the spring. This can kill
the tree.
8 Plant growth hormones are absorbed across the leaf surface and translocated to other parts
of the plant, including the roots.

Practice questions (pages 190–1)


1 a) The cell is hollow, with just a cell wall and no cell contents inside. [1]
b) The distance moved by the bubble in a minute would be measured in millimetres. The
volume of water this represents would be calculated using the formula for the volume of a
cylinder. The internal radius of the capillary would be squared and multiplied by π to give its
cross-sectional area. This would then be multiplied by the distance moved to give the volume
of water taken up in cubic millimetres. [3]
c) In humid conditions, the water potential gradient between air and leaves would be less steep
so transpiration would occur at a lower rate. [2]
d) 0.87 mm3 minute–1 [1]

2 a) i) So that water could leave the test tubes only by transpiration through the leaves and
not just by evaporation from the water surface into the air. [1]
ii) Ten leaves in each group provide repeat measurements to check for precision and to
allow a mean value to be calculated. [2]
b) i) Most of the water lost from leaves is lost via stomata. When stomata are blocked by
petroleum jelly, transpiration is reduced. More of the water is lost through stomata on
the lower surface, where the majority of stomata are found. Some transpiration occurs
through the waxy cuticle because even when both surfaces are covered in petroleum
jelly; some water is still lost. [4]
ii) Group D was to indicate the volume of water lost from test tubes containing leaves with
no petroleum jelly, with which the other values could be compared. [2]

3 a) i) The diameter of the trunk fluctuates daily, reaching its lowest value at the end of the
day and its highest value at the end of the night. [3]
ii) The tension built up during transpiration in the columns of water tends to make the
xylem vessels slightly narrower. The diameter of a tree trunk is reduced when the tree is
transpiring during the day, because the tension in the xylem is sufficient to pull in the
walls of the many vessels just a little. At night, when transpiration stops, the effect is
reversed. [4]
b) i) X = leaves, Y = stem [2]

© Pauline Lowrie and Mark Smith 2015


10 Mass transport in plants Answers

ii) It does support the theory, because as water molecules evaporate from part X, they
drag other molecules of water behind them because water molecules are polar, or
charged, and they are attracted to each other. This is cohesion. Because Y is narrow, the
column of water inside does not break, and water is pulled up. The pulling force is the
tension. [4]

© Pauline Lowrie and Mark Smith 2015


11 Genetic diversity Answers

Test yourself on prior knowledge (page 192)


1 Individuals in a population may look different to one another either because of the genes
they have inherited or the environment they have been exposed to or a combination of
both.
2 A mutation is an accidental change in a gene
3 If harmful bacteria become resistant to all or most antibiotics, it will become impossible to
treat serious bacterial infections.

Test yourself (page 197)


1 n represents haploid cells.
2 During meiosis, gametes with different combinations of alleles are formed, which
contributes to variation.
3 Assuming that just independent assortment took place, the number of genetically different
children one couple could potentially produce is 223 × 223 = 70 368 744 000 000. This is
because there are 223 possible combinations of chromosomes in sperm cells and 223
possible combinations of alleles in egg cells and any one sperm cell may fertilise any one
egg cell.

Test yourself (page 200)


4 If the mutation occurred in an intron it would not lead to a change in the encoded protein.
5 Yellow-shelled snails are less easily spotted among yellow-green grass stems than pink-
shelled snails. If song thrushes find and eat pink-shelled snails more easily, the yellow-
shelled snails will be more common.
6 An advantageous allele is one that gives the organism an increased chance of survival.
7 Mutations are rare events in the first place and since mutations alter characteristics that are
probably quite well adapted already, improving on them is unlikely.
8 When A is deleted from AGA, all of the remaining bases to the right move one place to the
left. AGA becomes AGU, which codes for serine instead of arginine. But all the other encoded
amino acids are also changed, because all the bases moving one place to the left alters all
the other codons too.
9 When U is substituted for A, AGU codes for serine rather than for arginine. But if G is
substituted for A, the codon would become AGG and this would still code for arginine.

© Pauline Lowrie and Mark Smith 2015


11 Genetic diversity Answers

Required practical 6: Use of aseptic techniques to investigate


the effect of antimicrobial substances on microbial growth
(page 202)
1 Nutrient agar is a jelly-like substance, containing nutrients, on the surface of which bacteria
can be grown. It has to be sterile so that only the required bacteria grow on it.
2 A broth is a liquid containing nutrients in which bacteria can be grown or cultured.
3 100 000 000
4 Aseptic technique means transferring the bacteria from the broth to the agar without
contaminating the plate with other micro-organisms and without allowing the bacteria in
the both to escape into the environment. Steps that would be taken would include using a
suitable antiseptic on the work surface, having a lit Bunsen burner in the vicinity, flaming
the pipette and spreader, flaming the mouth of the culture bottle and only removing the
Petri dish lid for the minimum time necessary.
5 So that the effect of the antibiotics can be compared.
6 So that the diffusing antibiotics do not overlap.
7 This is where no bacteria grow because they have been killed by the antibiotic.
8 There are no discs where there is no clear zone at all.
9 The one with the smallest clear zone.
10 Measuring right across the clear zone from one side to the other might be easier than
judging the centre of the paper disc.
11 Some of the clear zones have a fuzzy edge so it is difficult to decide where to measure to.
12 The table should have suitably labelled columns for antibiotic (number), diameter (mm)
and radius (mm; calculated by halving the diameter), displayed left to right in that order.
The radius values should have the same number of significant figures as the diameter
values.
13 The one with the largest radius.

Test yourself (page 203)


10 Mutations occur randomly so mutations that gave antibiotic resistance could have occurred
in these ancient bacterial populations whether or not they were exposed to antibiotics.
11 The modern cheetah population are the descendants of a small number of cheetahs that
survived some sort of ecological disaster in the past. This means that the modern
population has low genetic diversity which reduces the possibility of cheetahs adapting to
changing environmental conditions. Their long-term survival is therefore threatened.
12 Modern crop plants have a low genetic diversity due to artificial selection. The ancestors of
these modern crop plants may have had alleles that might prove advantageous to crop

© Pauline Lowrie and Mark Smith 2015


11 Genetic diversity Answers

plants in the future. Keeping their seeds means that they can be grown and crossed with
modern crop plants should these alleles need to be re-introduced.

Test yourself (page 205)


13 Any sensible factor, such as quality of nutrition or exposure to childhood illnesses.
14 Samples should be selected randomly because that avoids bias and helps to ensure that the
samples are representative of the whole population.
15 Although the mean height of both samples is the same, the standard deviations indicate that
there is more variation in the heights of group B. The heights in group A are more closely
clustered together.

Practice questions (pages 207–9)


1 a) [4]

Adenine Base that pairs with thymine in the DNA molecule

Locus The fixed position on a strand of DNA where a


particular gene is found.

Histone Protein associated with DNA in a eukaryotic


chromosome

Intron A sequence of nucleotides within a gene that


does not code for a polypeptide.

b) i) Both pairs have two chromosomes the same size. Both chromosomes of one of the pairs
carries the same genes in the same loci. [2]
ii) The chromosomes are visible as double structures. The members of each homologous
pair are aligned next to one another. [2]
c) Because crossing over has occurred between the members of some of the homologous pairs. [2]

2 a) There is a positive correlation between antibiotic use and development of resistant bacteria.
However, just having a correlation does not prove a causal link. Also, the study only looked at
middle ear infections in children and the antibiotic use does not increase very much over the
period of study, but antibiotic resistance in bacteria increases at a much greater rate. [5]
b) The sequence of events should include, in order: random mutation, selective advantage,
differential survival and an increase in the frequency of the resistance allele. [4]

3 a) i) A gamete is a cell specialised for sexual reproduction. [1]


ii) A gamete is haploid whereas a normal body cell is diploid. [2]

© Pauline Lowrie and Mark Smith 2015


11 Genetic diversity Answers

b) i) In a clockwise direction, the first three boxes are 47, the last one is 94 [2]
ii) M should be between the diploid stage of life cycle and spores. [1]
c) They have different combinations of maternal and paternal chromosomes and different
combinations of alleles [4]

4 a) The data shows that some intelligent parents do have intelligent children but intelligence is
not measured in a reliable way, we do not know what he means by ‘brilliant’, ‘scientific
ability’ or ‘normal’. In addition, no females are identified as intelligent which suggests bias
and if intelligence is inherited it must be inherited from both parents and not just the father.
Also, intelligent parents may provide an environment that stimulates their children, so this
could also be evidence for environmental factors having an influence. Finally, the sample size
is very small to base a conclusion on. [5]
b) Monozygotic twins genetically identical, dizygotic twins have genetic differences. Since the
points are all nearer the line among monozygotic twins, it suggests a strong genetic influence.
All sets of twins experience similar home environment but neither set of points are all on the
line, so there must be some environmental influence. [4]

5 a) The low number of original colonists created a founder effect leading to low genetic diversity
in the population. One or a few settlers had the Huntington’s allele and because they were a
relatively large proportion of a small initial population, they passed the allele on to many of
their descendants. [3]
b) One/some maggot flies undergo a mutation giving them an allele that allows them to feed on
apples. This is an advantage since it gives them a better food supply. Maggot flies that can
feed on apples survive longer and pass on the advantageous alleles to their offspring. This
leads to an increase in the frequency of the apple feeding allele in the population. [3]

© Pauline Lowrie and Mark Smith 2015


12 Species and taxonomy Answers

Test yourself on prior knowledge (page 210)


1 Any familiar two, such as Plants, Animals or Fungi
2 The components of a DNA nucleotide are deoxyribose, a phosphate group and one of the
organic bases adenine, cytosine, guanine or thymine.
3 Adenine is complementary to thymine and cytosine is complementary to guanine.
4 140
5 Functional RNA is RNA that is not translated into proteins.

Test yourself (page 217)


1 Lion and leopard both have the same generic name, Panthera, so they are more closely
related to each other than to the clouded leopard.
2 A horse and a donkey are different species because the mule is infertile. If cells in a mule
cannot undergo meiosis, mules cannot produce gametes and so they are unable to
reproduce.
3 A hierarchy is a series of groups in which smaller groups are placed within larger groups
with no overlap between them.
4 Phylogenetic classifications are based on evolutionary origins, and they show the
evolutionary relationships between taxa.

Activity: Another way of looking at differences in DNA:


comparing mRNA sequences (page 220)
1 No, based on this evidence, the tree shrew is not as closely related to humans as other non-
human primates in terms of its interleukin-7 mRNA base sequences.
2 Neither the rat nor the pig are primates, but the evidence suggests that they more closely
related to primates than many other mammals.

Test yourself (page 222)


5 The primary structure of proteins can differ in terms of which amino acids are present and
the sequence they are arranged in (they can also differ in terms of the number of amino
acids).
6 Not all organisms have haemoglobin.
7 DNA base sequences, RNA base sequences and the amino acid sequences of proteins.

© Pauline Lowrie and Mark Smith 2015


12 Species and taxonomy Answers

Practice questions (pages 223–4)


1 a) [2]

Domain Eukarya

Kingdom Plantae

Phylum Anthophyta

Class Magnoliopsida

Order Asterales

Family Compositae

Genus Taraxacum

Species officinale

b) They would have to carry out a breeding experiment to find out if they can reproduce to form
fertile offspring. They would need to cross-pollinate individuals of the two types of dandelion,
collect some of the seeds that form and grow them. They would then need to cross pollinate
some of these offspring and see if they also produced seeds. [2]

2 a) i) Cytochrome c is found in a much wider range of organisms than haemoglobin. [1]


ii) The number of amino acid differences between monkeys and humans is far lower than
the number of differences between pigs and humans. [2]
b) No, the data show that they both happen to have the same number of amino acid differences
between themselves and humans rather than showing that they have no differences between
each other. [2]

3 a) By chance, one of the rabbit’s many B cells might have receptors with a shape exactly
complementary to the shape of the injected snake albumin. If so, the receptors bind to the
albumin molecules. These B cells divide rapidly by mitosis to produce a large number of
daughter cells forming a clone. The majority of cells in the clone become plasma cells, which
release anti albumin antibodies. [4]
b) i) B. corruta and B. atropos are more closely related to each other than they are to
B. heraldica. [2]
ii) By measuring how much precipitate is formed as a result of the antibody–antigen
reaction. [3]

© Pauline Lowrie and Mark Smith 2015


12 Species and taxonomy Answers

c) i) Different rabbits may show different levels of immune response to the injected albumin. [1]
ii) Because the genes for albumin in the different species of snake may contain differences
in introns which are not translated into differences in their albumin molecules. [1]

© Pauline Lowrie and Mark Smith 2015


13 Biodiversity within a community Answers

Test yourself on prior knowledge (page 225)


1 A group of populations forms a community.
2 Any suitable example; suitable examples might be a barn owl adapted for nocturnal
hunting or a cactus adapted for an arid environment.
3 One might grow leaves above the other and compete for light or their roots might
overlap so they compete for water in the soil.
4 Animals are dependent either directly or indirectly on plants for food.

Test yourself (page 230)


1 A larger wood contains more tree species, so offers a wider range of food sources. A
larger wood offers larger quantities of food, which may support larger bird species in
addition to smaller ones.
2 By standing in one place for the same amount of time at the same time of day and
counting the number of species seen and heard. This should be repeated in the centre
of the wood and at the edge of the wood.
3 The tree diversity in a willow plantation would be lower due to the high density of the
willow planting. Willow trees would out-compete most other trees for light before they
became established.
4 At the outfall, the conditions are harsher (lower oxygen concentration, increased
turbidity) and fewer species are adapted to survive them. As the effluent is diluted
further from the outfall, conditions become less harsh and a wider range of less
specialised species can survive.

Activity: The effect of autumn sowing of cereal on rooks


(page 233)
1 In June, July and August
2 a) Spilt grain
b) Soil-living invertebrates
3 Four months out of twelve, or a third
4 South-east England is where a large amount of cereal is grown. The table shows that a
switch to autumn sowing of these cereals would reduce the food available to rooks,
resulting in a decrease in numbers.

Test yourself (page 234)


5 High milk yields can only be obtained by growing a mixture of ryegrass and white
clover and these species will only grow on well-drained and fertilised land.

© Pauline Lowrie and Mark Smith 2015


13 Biodiversity within a community Answers

6 Regular use of fertiliser encourages just two species which out-compete any others so
the diversity of plants remains low.
7 Fewer plant species means a smaller range of food sources and habitats for insects, so
fewer species of insect can survive.
8 The advantage of autumn sowing is the longer growing period which gives a higher
yield. But the disadvantage is that birds such as rooks cannot feed in arable fields
during the growing period. The proportion of the year when they can feed in autumn
sown fields is therefore much smaller.

Activity: The effect of organic farming on butterflies


(page 234)
1 So that, as far as possible, it was not the presence or absence of landscape features that
was causing the changes in species richness or abundance.
2 To compare the effect of organic versus conventional farming in both types of
landscape.
3 Organic farming practices increase the butterfly species richness and the butterfly
abundance more in uniform landscapes than in mixed landscapes.
4 Because in uniform landscapes there are fewer marginal habitats in between the arable
fields that might otherwise support higher butterfly numbers or more species of
butterfly. So organic farming makes a significant contribution to species richness.
5 Using fewer pesticides would kill fewer butterflies so increase abundance. Using fewer
inorganic fertilisers would prevent some plant species dominating others, leading to a
greater diversity of plants, which would in turn provide a greater range of food sources
to support different butterfly species, which would increase the species richness.
6 No, because the abundance of each separate species would be required and the graph
only gives the total abundance.

Test yourself (page 236)


9 The use of many pesticides is not allowed and neither is the use of inorganic fertilisers.
This ensures a higher diversity of both plants and insects.
10 Pollinators are vital for the pollination and therefore yield of many commercial crops.
11 Neonicotinoids protect crop plants from pests immediately from germination because
they are absorbed by the seeds and translocated.
12 Neonicotinoids reduce their ability to fly and to navigate to find food.

© Pauline Lowrie and Mark Smith 2015


13 Biodiversity within a community Answers

Practice questions (pages 237–8)


1 a) As the length of hedge increases, so does the number of bird species. There is a positive
correlation between the length of the hedge and the number of bird species found. [2]
b) A longer hedge contains more plant species which provides a wider range of food
sources supporting different bird species. A longer hedge also contains a larger variety of
nesting sites suitable for a wider range of bird species. [2]
c) Calculating an index of diversity requires the abundance of each species. Counting the
number of bird species would have been less time consuming and faster than finding the
abundance. But just one individual of a species present counts the same as many
individuals so simply using species richness does not take any account of the evenness of
the bird community. [3]
d) The highest bird species richness was found in hedges around organic fields and the
lowest in hedges surrounding conventional fields. But there is relatively little difference
in the general pattern of points for hedges around organic and conventional fields and
many hedges around organic fields have far fewer bird species than those of the same
length around conventional fields. [4]

2 a) i) The extent of the area in which the students made the observations (from just one
point in each wood, or along a transect of the same length in each wood), so that
the results for the two woodlands are comparable. [2]
ii) The two students might have different bird-recognition skills, leading to fewer bird
species being identified by one student compared to the other. [2]
b) i) Σn(n − 1) = 7(7 − 1) + 2(2 − 1) + 1(1 − 1) + 3(3 − 1) + 2(2 − 1) + 4(4 − 1) + 4(4 − 1)
+ 2(2 − 1) + 12(12 − 1) + 6(6 − 1) = 241
So d = 43(43 − 1)/241 = 1806/241 = 7.49 [2]
ii) An index of diversity takes account of relative abundance as well as species
richness. [1]

3 a) The mix of different species at a range of scales, from a small patch of meadow to a
whole country. [2]
b) The graph shows changes in total abundance as a percentage of the starting abundance,
not change in population size. There might have been far more moths than butterflies at
the start of the study period. There are different numbers of species of butterfly and
moth present and the populations of some species may have increased while others
decreased. [4]
c) The number in 1998 divided by the number in 1993 multiplied by 100. [3]

© Pauline Lowrie and Mark Smith 2015


14 Energy transfer Answers

Test yourself on prior knowledge (page 239)


1 Any suitable two substances, e.g. cellulose, lignin, protein
2 Any two energy-requiring processes, e.g. active transport, DNA synthesis, cell division,
protein synthesis
3 ATP synthase
4 Heat

Required practical 7: Use of chromatography to investigate the


pigments isolated from leaves of different plants (page 243)
1 A solvent is a liquid able to dissolve other substances. In this situation, the plate is placed in
a small volume of chromatography solvent. The solvent slowly moves up the plate. As the
solvent moves, the different pigments dissolve in it and are carried up with it, at different
rates.
2 So that the origin spot is not simply washed off the plate into the solvent.
3 By adding a series of small drops and drying the spot in between each addition.
4 The pigments are, in descending order from the top of the plate: carotene, xanthophyll,
chlorophyll a, chlorophyll b.
5 Not measuring to the same place, such as the centre, for each spot.

Required practical 8: Investigation into the effect of a named


factor on the rate of dehydrogenase activity in extracts of
chloroplasts (page 245)
Tube 1: the colour of the solution returns to the original green colour of the chloroplast extract.
This is because photoionisation of chlorophyll molecules occurs in the light and electrons are
transferred to DCPIP molecules, reducing them and causing them to become colourless.
Tube 2: the solution remains blue. Because the tube is completely wrapped in foil, the
chloroplast extract receives no light so no photoionisation occurs and no electrons are
transferred to DCPIP molecules.
Tube 3: the solution remains blue. Boiling the chloroplast extract denatures proteins in the
electron transfer chains, so no electrons are transferred to DCPIP molecules. This tube shows
that DCPIP does not become colourless when exposed to light.
Tube 4: the solution remains blue. Boiling the chloroplast extract denatures proteins in the
electron transfer chains, so no electrons are transferred to DCPIP molecules. This tube shows
that DCPIP does not become colourless in the dark.
Tube 5: the solution remains green because there is no DCPIP present. This tube shows that the
extract does not change colour in the light.

© Pauline Lowrie and Mark Smith 2015


14 Energy transfer Answers

Tube 6: the solution remains green because there is no DCPIP present. This tube shows that the
extract does not change colour in the dark.

Test yourself (page 247)


1 Photoionisation is the process by which a chlorophyll molecule becomes positively charged
as a result of losing two electrons when it absorbs light.
2 a) They replace those lost in the first chlorophyll molecule.
b) They are transferred along an electron transfer chain and are used to produce reduced
NADP.
c) They are transferred along an electron transfer chain, resulting in ATP production.
3 Triose phosphate
4 The RuBP reacts with carbon dioxide and is converted to GP.

Test yourself (page 252)


5 Some light energy is reflected from leaves, some is transmitted and some is used to
evaporate water in transpiration.
6 It would be heated in an oven to constant mass without causing any combustion.
7 NPP = GPP − R
8 g m–3 day–1 or kJ m–3 day–1
9 Leaves, fruits and other parts of the trees fall to the ground at different times of the year,
branches may break off in high winds and eventually the whole tree dies and falls over.

Required practical: Investigation into the effect of a named


variable on the rate of respiration of cultures of singled-celled
organisms (page 257)
1 A suitable graph would have time on the x-axis, ethanol concentration on the y-axis and
show four labelled curves, one for each glucose concentration.
2 Initial rate at 10% glucose is 0.57 g dm−3 h−1, at 20% glucose is 0.50 g dm−3 h−1, at 30%
glucose is 0.28 g dm−3 h−1 and at 40% glucose is 0.16 g dm−3 h−1.
3 As the glucose concentration is increased, the initial rate of ethanol production decreases.
The decrease is smaller at lower glucose concentrations and becomes larger as glucose
concentration is increased.
4 Total ethanol production rises with increasing glucose concentration, reaching a peak at
30% glucose and then declining.
5 Increasing the glucose concentration provides the yeast with more respiratory substrate, so
it might be expected that the rate of anaerobic respiration would increase with increasing
glucose concentration. However, as the glucose concentration is increased, the water

© Pauline Lowrie and Mark Smith 2015


14 Energy transfer Answers

potential of the broth is decreased. This exerts an osmotic stress on the yeast, reducing the
efficiency of anaerobic respiration. At 30% glucose concentration, the total ethanol
production is highest because there is more glucose available so although the rate of
anaerobic respiration is lower, the total is eventually higher. Above 30% glucose
concentration, the efficiency of anaerobic respiration is reduced so much that the total
ethanol production is also reduced.

Test yourself (page 257)


10 In the cytoplasm
11 Pyruvate, reduced coenzyme and ATP; pyruvate is used in the Krebs cycle, reduced
coenzyme transfers electrons to electron transfer chains and ATP is used by the cell for
energy-requiring processes.
12 Substrate-level phosphorylation is the production of ATP linked to the reaction of a
substrate molecule. Oxidative phosphorylation is production of ATP by the electron transfer
chain in aerobic respiration, using oxygen as the final electron acceptor.
13 It is converted to lactate.
14 In swampy conditions, there is little oxygen in the mud so the roots of the rice will respire
anaerobically, converting pyruvate to ethanol.

Test yourself (page 263)


15 N = I − (F + R)
16 There is chemical potential energy in any undigested biological molecules in the faeces.
17 Since energy is lost as heat from the organisms at each trophic level, there is usually
insufficient energy remaining to support further trophic levels.
18 Fish do not maintain a high body temperature like chickens, so do not need to carry out
respiration to stay warm, so less energy is lost as heat. Farmed fish eat pelleted food which
is highly digestible, so very little energy is lost via their faeces compared to free-range
chickens, which eat a mixture of food they find themselves.
19 Being kept in a warm environment means that broilers lose less heat to their surroundings
so they do not need to carry out as much respiration to remain warm.

Practice questions (pages 264–6)


1 a) In the cytoplasm [1]
b) i) Pyruvate [1]
ii) Carbon dioxide [1]
c) The link reaction [1]
d) Substrate-level phosphorylation [1]

© Pauline Lowrie and Mark Smith 2015


14 Energy transfer Answers

e) Reduced coenzymes transfer their electrons to electron transfer chains, which results in the
production of ATP by oxidative phosphorylation. [2]
2 a) Reduced NADP [1]
b) Triose phosphate [1]
c) i) Rubisco [1]
ii) Glycerate 3-phosphate [1]
d) The concentration falls and then plateaus as it reacts with carbon dioxide until remaining ATP
and reduced NADP from the light-dependent reactions are used up. [2]
3 a) i) Primary consumer (herbivore) [1]
ii) A secondary consumer [1]
b) i) It remained relatively similar until 1970, after which it began to fall. [2]
ii) They changed the proportions of different prey species eaten, consuming more
food at trophic level 3 rather than 4. [2]
c) It has increased as their diet has included more animal material in addition to grass,
so they are no longer entirely herbivorous. [2]
4 a) Biomass can be measured as mass of carbon or dry mass of tissue per given area. [1]
b) i) Heat the heather samples in an oven at around 90 °C to evaporate the water
they contain. [2]
ii) Continue heating the samples until three successive mass measurements are
the same. [1]
iii) Plant material can contain varying amounts of water depending on the
environmental conditions prior to sampling. [1]
c) The mean annual increase in dry biomass can be calculated by finding the gradient of
the line. The construction lines on the graph show that the biomass increases by
640 g in 5 years. This is an increase of 128 g yr−1. [2]
d) The points do not all lie on the curve of best fit. [1]
e) Abiotic factors other than light, such as rainfall and temperature, can also affect
growth/biotic factors, such as the amount of grazing by animals can also affect biomass. [1]
f)
chemical potential energy increase in an area of heather plants in a year
Efficiency= × 100%
light energy falling on this area of heather plants in a year

chemical potential energy in 1 g of heather × increase in biomass of 1 m2 heather in a year


Efficiency= × 100%
total amount of light falling on 1 m2 of heather moorland in a year

27.3 × 128
= × 100%
1415000
= 0.2% [3]

© Pauline Lowrie and Mark Smith 2015


14 Energy transfer Answers

Stretch and challenge questions


5 You should refer to the role of chemoautotrophs and explain how these act as producers in food
webs. You should then go on to explain the transfer of energy through the food web.
You may choose to research the different kinds of bacteria found in these ecosystems.
6 Answers should include a clear account of CAM and C4 photosynthesis, and details of how they
differ from ‘standard’ photosynthesis. They should relate these pathways to the kinds of plant
that have these alternative pathways, and the environment they live in. As an extension you
should be able to explain how these pathways are an advantage in certain environments, but
might not be in others. It is also possible to examine the benefits of growing C4 plants as crops in
tropical areas.

© Pauline Lowrie and Mark Smith 2015


15 Nutrient cycles Answers

Test yourself on prior knowledge (page 267)


1 Warm, damp, aerobic conditions
2 Fungi and bacteria
3 Nucleic acids
4 ATP has three phosphate groups in a chain, attached to a molecule of ribose.

Test yourself (page 271)


1 Nutrient recycling is vital because there is only a limited supply of nutrients and so they
must be recycled, otherwise they would all become locked up in biological material.
2 Decomposition is the process in which the biological molecules in dead material and waste
products are digested, producing carbon dioxide, water and inorganic ions.
3 Saprobionts are organisms that secrete digestive enzymes onto the dead remains of other
organisms, digest the biological molecules in these dead remains, and then absorb some of
the products of this digestion.
4 Saprobionts hydrolyse biological molecules by secreting enzymes into their surrounding
environment. Some of the products of hydrolysis are then absorbed by the saprobionts, but
many remain in their surroundings and may then be absorbed by other organisms.

Test yourself (page 276)


5 The nitrogen cycle and the carbon cycle both involve fixation of a gas from the atmosphere,
they both involve the element being passed between trophic levels by feeding and they both
involve decomposition.
6 The phosphorous cycle does not involve a gas in the atmosphere; the phosphorous cycle
involves the element spending a very long time in sedimentary rocks.
7 Nitrification is the two-step oxidation of ammonium ions, firstly to nitrite ions and then to
nitrate. It is important to plants because it makes nitrate ions available for them to absorb.
8 Mycorrhizae are mutualistic associations between plant roots and beneficial fungi. They
increase the surface area of the root system for the uptake of ions and water from the soil.

Test yourself (page 279)


9 a) When crops are harvested, some or all of the biomass is taken away and the chemical
elements in the biomass have to be replaced in the soil otherwise the next crop would be
nutrient limited.
b) Dairy cattle and sheep feed on the grass but they are then removed from the fields so the
chemical elements they have gained by eating the grass are taken away and have to be
replaced.

© Pauline Lowrie and Mark Smith 2015


15 Nutrient cycles Answers

10 Natural fertilisers decompose to release their nutrients, which takes time, but artificial
fertilisers just dissolve and the nutrients are available immediately.
11 The maize will grow and generate some yield without any fertiliser being used.
12 170 kg per hectare, because this gives the highest yield of maize.

Activity: The effect of phosphate concentration on the growth


of algae using data from an experiment (page 283)
1 10 000 times greater
2 a) The algal population in the lake decreases sharply between October and December,
whereas the algal population in the tube increases slightly. Between December and
March they both increase, slower at first in the lake and then more rapidly from mid-
February.
b) Inside the tube, the algal population were not limited by phosphate so they were able to
increase more rapidly than those in the lake, which were limited by phosphate
concentration.
3 The water inside the tube may have had a different temperature to that outside in the lake.
4 Other factors might influence algal population growth in addition to phosphate availability.

Test yourself (page 283)


13 Leaching is when soluble ions dissolved in soil water drain through the soil into aquatic
ecosystems. It is a risk to aquatic ecosystems because it can lead to eutrophication, which in
turn can lead to the death of aquatic invertebrates.
14 When artificial fertilisers dissolve they provide a lot of nutrients at once, more than plants
can absorb immediately, leaving the excess in the soil from where they can be leached.
Natural fertilisers release nutrients slowly so they are absorbed by plants as fast as they are
released.
15 If the concentration of nitrate and phosphate ions in the water is increased it promotes the
growth of algae, which can coat the leaves of other aquatic plants and make the water
cloudy. Much less light reaches some of the algae and other aquatic plants so they
photosynthesise too slowly and die. The dead plant material is decomposed by saprobionts
in the water, especially bacteria. They respire aerobically and use of much of the oxygen in
the water. This can result in less tolerant aquatic invertebrates and fish dying.
16 Sampling all the lakes in the same year means that other factors that might have affected
algal growth, such as temperature and light availability, would have been the same. This
ensures that the only factor affecting algal growth is the nutrient concentration in the
water.

© Pauline Lowrie and Mark Smith 2015


15 Nutrient cycles Answers

Practice questions (pages 284–5)


1 a) i) Increased algal growth reduces the light able to reach other aquatic plants so they
are unable to photosynthesis enough to survive. [2]
ii) Dead plant material is decomposed by bacteria, which use up the oxygen for aerobic
respiration. [2]
b) Animal species that are intolerant of oxygen depletion will be unable to survive, leaving a
community of just a few tolerant species. [2]
2 a) Phosphate ions may be absorbed directly from their food. Biological molecules such as
nucleic acids in their food are digested to smaller phosphate-containing molecules, such as
nucleotides, and absorbed. [2]
b) Nucleotides and phosphate ions are used to produce nucleic acids and ATP. [2]
c) When organisms produce faeces or die the phosphorus-containing substances in their
faeces or tissues are digested by saprobiotic bacteria, releasing phosphate ions, which
can then be taken up again by plants. [2]
3 a) To reduce the risk of damage to aquatic ecosystems [1]
b) Saprobionts decompose the plant material, secreting enzymes to hydrolyse large biological
molecules into smaller molecules and releasing nutrients. [3]
c) Because they will decompose slowly, enabling plants to take up the nutrients as they
are released. [2]
d) To synthesise ATP and to synthesise DNA and RNA [2]
4 a) The leaf litter will contain variable amounts of water, so determining dry mass would
ensure that only the mass of biological material was being measured. [2]
b) The fir leaves lost the largest percentage of biomass in a year, so they have the highest
rate of decomposition, but have the lowest lignin content. Saprobionts are able to more
easily digest the fir leaves because they contain less lignin. Beech leaves contain more
lignin, so saprobionts are less easily able to digest them and so they have a slower rate
of decomposition. [3]
c) Beech leaves have a C:N ratio of 72.5:1 whereas fir leaves have a C:N ratio of 30.5:1. Fir
leaves have the closest ratio to that required by microorganisms for respiration and protein
synthesis. Bacterial growth will be faster on fir leaves and this relates to their higher rate of
decomposition. [3]

Stretch and challenge question


5 You should explain what mycorrhizae are and how they can be an advantage to some plants.
Ideally you will refer to mutualism, or at least symbiosis. You should research the scientific work
referred to and evaluate the evidence provided by these scientists. You should then refer to

© Pauline Lowrie and Mark Smith 2015


15 Nutrient cycles Answers

orchid conservation and how scientists ensure that mycorrhizae are propagated alongside
orchids.

© Pauline Lowrie and Mark Smith 2015


16 Response Answers

Test yourself on prior knowledge (page 286)


1 Receptors are specialised cells that detect stimuli; any two suitable suggestions, e.g. touch
receptors, light receptors, pain receptors.
2 Rapid reflex responses can protect animals from harm.
3 A synapse is a junction between two nerve cells. The cells do not actually touch. There is a
tiny gap between them, across which chemical substances diffuse.
4 Plant shoots that grow towards light enable leaves to absorb more light for photosynthesis
and plant roots that grow down into the soil are able to provide better anchorage and
absorb more nutrients from deeper in the soil.

Required practical 10: Investigation into the effect of an


environmental variable on the movement of an animal using
either a choice chamber or a maze (page 292)
1 Woodlice stop moving/move less in humid conditions.
2 This response reduces water loss from the respiratory surface.
3 It suggests that this response is a kinesis.
4 Percentage change between experiments A and C = 113 − 17/113 x 100 = 84.9% fewer still
moving. There is a larger reduction in the percentage of woodlice still moving around after
15 minutes if the woodlice have been kept in the dark for several days before the
experiment. This suggests that they are more active in the light and that if they are kept in
the light for several days beforehand, they remain more active during the experiment. The
graph should be a bar chart.
5 Woodlice respond more actively to humidity if they are in the light. If they are exposed to
dry conditions during the day when they are more likely to dehydrate, they are more likely
to move into more humid conditions under stone, logs and so on. When they are under such
objects, they tend to stop moving which means they remain in more humid conditions.

Test yourself (page 296)


1 The components of a three-neurone reflex arc are a receptor, a sensory neurone, a relay
neurone, a motor neurone and an effector.
2 This is a negative phototaxis because it is a directional movement away from the stimulus.
3 No, because the student can only see how many woodlice ended up in the light or the dark,
not how they got there.
4 Plant responses are growth responses. The stem or root bends because more growth occurs
on one side compared to the other.

© Pauline Lowrie and Mark Smith 2015


16 Response Answers

5 Amyloplasts are dense organelles that sink to the bottom of the cells they are in because of
the force of gravity. If the orientation of the root is changed, the amyloplasts sink to what
has become the bottom of the cells and the cells are able to detect the new position of the
amyloplasts.

Test yourself (page 302)


6 A generator potential is an electrical potential difference created in a receptor in response
to a stimulus.
7 They open in response to pressure, allowing faster exchange of sodium ions.
8 It means that the sinoatrial node generates waves of electrical activity without a stimulus
from the nervous system.
9 The AVN detects the wave of electrical excitation passing over the atria and conducts
electrical activity to the bundle of His after a short delay, which allows time for the
ventricles to fill with blood.
10 In the wall of the aorta close to the heart and in walls of the carotid arteries that pass
through the neck to the brain.

Test yourself (page 304)


11 Rod and cone cells have differently-shaped outer segments. Most of the cone cells are near
the centre of the retina at the fovea
12 Rod cells are more sensitive to light than cone cells because the optical pigment within rods
is broken down in dim light whereas that within cones is only broken down in bright light.
13 Visual acuity describes how far apart two spots of light must be to be seen as separate. It
indicates the level of detail, or the resolution, of the image detected.
14 If more receptor cells are connected together to one sensory neurone, the visual acuity at
that point on the retina will be lower because the brain will not be able to distinguish
between a spot of light falling on any one of the receptors.
15 When combinations of the three types of cone are stimulated we perceive the range of other
colours in the visible spectrum.

Practice questions (pages 305–6)


1 a) i) 8 (minutes) [1]
ii) A logarithmic scale [1]
iii) It allows values of a range of orders of magnitude to be plotted on the same axis. [1]
b) i) A kinesis [1]
ii) They do not move directly towards or away from the light, but the brighter
the light the more they randomly move around. [1]

© Pauline Lowrie and Mark Smith 2015


16 Response Answers

c) The response will have the effect of keeping the lamprey larvae buried in the mud at the
bottom of the river because when they are in darker conditions they stop moving as much.
If they emerge from the mud, they move around randomly until they become buried again. [2]
2 a) A and B show that the shoot tip makes the same amount of IAA in the dark as it does
in light. Light does not increase the production of IAA. [1]
b) The thin glass cover slip stops the diffusion of IAA from one side of the shoot tip to
the other. [1]
c) D shows that light from one direction causes some IAA to diffuse from the lit side to the
shaded side before diffusing downwards. C shows that the higher concentration of IAA
on the shaded side is not due to more being made on the shaded side. If sideways diffusion
is prevented, the same concentration diffuses down each side. [2]
d) The shoot would bend towards the light because the higher IAA on the shaded side
would cause more cell elongation on that side of the shoot. [3]
3 The SAN is a group of myogenic muscle cells. They contract regularly and each time they
contract they cause a wave of electrical excitation to spread over the walls of both atria,
causing atrial contraction. The wave of electrical excitation cannot pass straight to the
ventricles because of a layer of insulating fibrous tissue. Instead, the AVN detects the wave of
electrical excitation as it passes and relays it to the bundle of His after a short delay. The delay
allows time for the ventricles to fill with blood. The wave of electrical excitation then moves
rapidly down through the Purkyne tissue and spreads out across the base of the ventricles,
causing ventricle contraction from the base upwards. [6]
4 a) X is the blind spot because there are no receptor cells present at that point.
Y is the fovea because there are just cone cells present at that point. [2]
b) 16:1 [1]
c) At 10 arbitrary units. Only the cones have optical pigments that are sensitive to specific
colours of light and there are more cones at this point. [3]
d) i) Receptors at Y have a lower sensitivity to light than those at Z. [1]
ii) This is because brighter light is required to break down the optical pigment and produce
a generator potential in cones whereas the optical pigment in rods is broken down in
only dim light. [2]
e) Light that falls on just one cone at Y will be detected as a single spot of light because each
cone is connected to a single ganglion cell in the optic nerve via one bipolar cell. If another
ray of light falls on the other cone the brain will be able to interpret this as two separate
spots. Region Y therefore has a higher visual acuity. If two rays of light fall on two separate
rods at Z the brain cannot interpret which of the rods has been stimulated because rods are
connected in groups to each ganglion cell. The brain interprets the two rays of light as one
single spot so region Z has a lower visual acuity. [3]

© Pauline Lowrie and Mark Smith 2015


16 Response Answers

Stretch and challenge questions


5 You should research the trichromatic theory of colour vision and the opponent-process theory.
You should evaluate both these theories and the evidence for them. You may describe the visual
pathways in the brain, and use these to explain the theories of colour vision.
6 The answer should include chemicals that mimic IAA in selective weedkillers, developing seedless
fruit and hormone rooting powder. You may examine the use of plant growth regulators in plant
tissue culture and in stimulating the growth of specific plant tissue types, e.g. root development.
You may go on to look at the role of ethane in fruit ripening, cytokinins in cut flowers and
numerous uses in crop production. As part of this, you should discuss the cost-effectiveness of
using these chemicals by comparing the cost of applying them to the improved yield.

© Pauline Lowrie and Mark Smith 2015


17 Nervous coordination Answers

Test yourself on prior knowledge (page 307)


1 Pacinian corpuscle, stimulated by pressure; rod cell, stimulated by light; or any other
suitable receptor.
2 An effector carries out a response.
3 Ions are charged particles and cannot pass across the lipid bilayer so they require carrier
proteins to be able to diffuse across membranes.
4 Cells make most ATP on the cristae of mitochondria during aerobic respiration. Although
they make some ATP during glycolysis, most is made during oxidative phosphorylation,
which involves electron transfer chains on the inner mitochondrial membrane.

Test yourself (page 312)


1 Downwards, from the cell body at the top to the branches at the bottom
2 Gated ion channels allow three sodium ions out of the axon for every two potassium ions
moving in. The membrane is also differentially permeable to sodium and potassium ions.
Potassium ions diffuse back across the membrane more rapidly than sodium ions. The
overall effect is to create and maintain the resting membrane potential.
3 When a stimulus causes the membrane potential to reach threshold, sodium ion channels
change their shape to open. This allows sodium ions to diffuse into the axon, depolarising it.
4 Sodium ion channels close while potassium ion channels open, allowing potassium ions to
diffuse out until the resting potential is restored.

Activity: Analysing an action potential (page 312)


1 A resting potential C repolarisation
B depolarisation D hyperpolarisation
2 Sodium ions diffuse into the axon, making the inside more positive than the outside,
depolarising the membrane.
3 Because threshold has to be reached before sodium channels open.
4 Increased permeability to potassium ions repolarises the membrane.
5 The change in sodium ion permeability is larger and lasts for a shorter time than the change
in potassium ion permeability.
6 It means the action potential is fast whereas recovery takes a little longer, helping to keep
impulses separate
7 36 mV
8 1.4 ms
9 1.6 ms

© Pauline Lowrie and Mark Smith 2015


17 Nervous coordination Answers

Test yourself (page 315)


5 The refractory period ensures that impulses remain separate from one another and do not
merge together.
6 The all-or-nothing principle is the idea that once threshold is reached an action potential is
triggered. The strength of the stimulus does not affect the size of the impulse.
7 Axon diameter, presence or absence of myelin, temperature.
8 When impulses jump from node to node in myelinated neurones.

Test yourself (page 320)


9 Because only the synaptic knob has vesicles containing neurotransmitter and only the post-
synaptic membrane has neurotransmitter receptors.
10 If acetylcholinesterase is inhibited, acetylcholine remains in the synaptic cleft and synaptic
transmission is continuous.
11 During temporal summation several impulses reach the synaptic knob in quick succession
so enough acetylcholine is released for the membrane potential to reach the threshold
value. During spatial summation, several impulses arrive simultaneously at different
synaptic knobs stimulating the same cell body so the cell body is depolarised enough for an
action potential to be generated.
12 It causes potassium ions to leave the post-synaptic neurone, thus preventing the threshold
stimulation needed to cause a generator potential.

Practice questions (pages 322–3)


1 a) i) Sodium ions
ii) Potassium ions [1]
b) When the threshold potential is reached, sodium ion channels open briefly, increasing
the permeability of the axon membrane. The sodium ion channels then close again,
reducing the permeability of the axon membrane. [2]
c) Depolarisation causes potassium ion channels in the membrane to open, increasing the
permeability of the axon membrane and allowing potassium ions to diffuse out. The
outward diffusion of positively charged ions helps to repolarise the axon membrane. [2]
d) i) The refractory period is the time following an action potential during which another
action potential cannot take place because the resting potential has not yet been
restored.
ii) The all-or-nothing principle is the idea that if a stimulus does not cause a neurone to
reach threshold potential, no action potential will occur, but if the stimulus causes

© Pauline Lowrie and Mark Smith 2015


17 Nervous coordination Answers

the neurone to exceed the threshold potential, the same action potential will occur
regardless of the size of the stimulus. [2]
2 a) i) Synaptic vesicle
ii) Acetylcholine [1]
b) X–Y. The synaptic vesicles are at X, so neurotransmitter will be released at that side of
the synaptic cleft and will diffuse towards Y. [1]
c) When an action potential arrives at the pre-synaptic membrane it stimulates calcium ion
channels to open. Calcium ions diffuse into the synaptic knob. This causes the synaptic
vesicles to move to the membrane and fuse with it. The acetylcholine is released into the
cleft and diffuses across to the membrane on the other side. When acetylcholine molecules
bind to receptor molecules the receptors change shape and allow sodium ions to diffuse
through, depolarising the post-synaptic membrane. [4]
d) Atropine would prevent the binding of acetylcholine to acetylcholine receptor molecules,
which would prevent sodium ion diffusion. The post-synaptic membrane would not be
depolarised. [2]
3 The cell-surface membranes of neurones contain sodium-potassium pumps. These pumps
actively move three sodium ions out for every two potassium ions moved in. There are
also protein channels in the membrane that allow these ions to diffuse back down their
concentration gradients. These channels are always open. However, the membrane is more
permeable to potassium ions than to sodium ions. This means that the potassium ions diffuse
back down their concentration gradient more rapidly than the sodium ions. The overall result
is that the outside of the axon membrane always has a slight excess of positive ions compared
to the inside. [5]
4 a) Myelinated means that the axon has fatty Schwann cells wrapped around it at regular
intervals. The axon is only exposed at the nodes of Ranvier. This means that the action
potential shows saltatory conduction. It jumps along the axon from node to node and is
conducted faster than if the axon were unmyelinated. [3]
b) This is spatial summation. There is not enough neurotransmitter released at synapse 1 to
cause the motor neurone to reach threshold potential. But if neurotransmitter is released at
both synapses together, there is sufficient neurotransmitter to reach threshold potential. [3]
c) The inhibitory neurotransmitter would make it more difficult for the motor neurone to reach
threshold, so even though action potentials arrive at 1 and 2 together, it may not cause an
action potential in the motor neurone. [3]

Stretch and challenge questions


5 You should be able to relate botox to botulinum toxin from Clostridium botulinum. There should
be a clear account of its interaction at the synapse. You could research its toxicity and the

© Pauline Lowrie and Mark Smith 2015


17 Nervous coordination Answers

different forms of the toxin that are available. You should then present a clear argument
weighing up the risks of using a potentially fatal toxin with its benefits, and safeguards when
using it. It is possible that you may also refer to therapeutic uses of botox/botulinum toxin.
6 You should give an account of the damage to myelin that occurs in multiple sclerosis (MS), and
relate this to the symptoms. A good account would show an understanding that MS can progress
differently in different people. You should then relate your understanding of the immune system
from year 1 of the course to examine the idea that MS is an autoimmune disease, as well as
evaluating alternative theories.

© Pauline Lowrie and Mark Smith 2015


18 Muscles and movement Answers

Test yourself on prior knowledge (page 324)


1 A pair of antagonistic muscles contract in opposite directions to one another across a joint.
2 Muscles can only cause effective movement if they pull on a structure, like bone, that does
not bend or shorten.
3 A motor neurone has a cell body at one end and a long axon surrounded by a series of
Schwann cells that wrap round and round the axon.

Test yourself (page 328)


1 D, because it must pull on the same bone in the limb as A, but in the opposite direction.
2 Tendons are lengths of tough connective tissue that join skeletal muscle to bone.
3 Fast fibres because eye muscles are used for frequent, rapid movements but do not remain
contacted for long.
4 There is a positive correlation between the mean percentage of slow fibres and mean
maximum oxygen concentration. This is because slow fibres remain aerobic during
contraction and so consume more oxygen compared to fast fibres.
5 A higher proportion of slow fibres in leg and arm muscles is an advantage for long-distance
running because they can maintain activity for long periods without becoming fatigued.

Test yourself (page 333)


6 The striations on a skeletal muscle myofibril are caused by alternating regions of
overlapping and non-overlapping protein filaments.
7 An actin filament consists of two chains of actin molecules wrapped around one another in
a double helix. Tropomyosin (and troponin) molecules are attached to the double helix.
8 The A band would stay the same size because it is the darker region containing myosin
filaments and the filaments themselves do not change length during contraction. The I band
would become smaller because it is the lighter region containing actin filaments between
which myosin filaments slide. This means the region containing just actin becomes smaller
during contraction.
9 ATP hydrolysis is required for the recovery stroke, where the myosin head returns to its
starting position ready to attach to another binding site on the actin filament.
10 Sarcoplasmic reticulum stores calcium ions, releases them into the muscle fibre cytoplasm
in response to an action potential and actively takes them back up again when the stimulus
ceases.

Activity: Muscle contraction (page 335)


1 One-tenth

© Pauline Lowrie and Mark Smith 2015


18 Muscles and movement Answers

2 38 ms
3 Calcium ions were being released from the sarcoplasmic reticulum.
4 Calcium ions separate from the troponin and are actively taken up again by the
sarcoplasmic reticulum. The tropomyosin molecules cover the binding sites on the actin
molecules.
5 The acetylcholine from each stimulus builds up. This is called summation.
6 The strength of a muscle contraction increases as more motor neurones are recruited and
more fibres contract.
7 Refractory period
8 It ensures that the heart always has a period to relax so heart contractions remain separate
and it cannot become continuously contracted.

Test yourself (page 337)


11 A: myelin sheath or Schwann cell; B: node of Ranvier.
12 Acetylcholine molecules are released at the neuromuscular junction, diffuse across the
synaptic cleft and attach to receptor molecules on the sarcolemma. The receptor molecules
change shape and allow sodium and potassium ions to diffuse through the sarcolemma.
Sodium ions diffuse in more rapidly than the potassium ions diffuse out. The excess of
sodium ions entering the neurone depolarises the sarcolemma.
13 The resting potential is restored because potassium ions diffuse out of the muscle fibre until
the potential difference returns to −70 mV. The fibre is then repolarised. The sodium and
potassium channels help to restore the balance by active transport.
14 Phosphocreatine is used in muscle fibres to produce ATP for a short period until more ATP
can be supplied from the mitochondria.
15 Any three differences from Table 5.2

Practice questions (pages 338–9)


1 a) i) S is actin, T is myosin [1]
ii) The X should be placed on the myosin head. [1]
iii) The arrow should point from right to left. [1]
b) The role of ATP is to provide energy for the recovery stroke. [1]
c) The myosin heads attach to the binding sites. The myosin molecules bend, pulling the
actin filaments along a little and releasing ADP and inorganic phosphate. An ATP molecule
attaches to the myosin head and it separates from the actin. The cycle then repeats again. [5]
3 a) i) Myosin
ii) Actin and myosin [1]

© Pauline Lowrie and Mark Smith 2015


18 Muscles and movement Answers

b) The distance would decrease; because the actin filaments slide between the myosin
filaments, so the regions of overlap increase whilst the overall distance between the
Z lines decreases. [2]
c) It would become smaller; because this is the region that contains just actin and when the
fibril contracts the actin slides between the myosin so the region containing just actin
is reduced. [2]
d) Sarcoplasmic reticulum stores calcium ions; when action potentials spread across the
sarcolemma and into the sarcoplasmic reticulum, the sarcoplasmic reticulum releases
calcium ions into the cytoplasm; where they initiate actinomyosin bridge formation. [3]
3 a) Acetylcholine [1]
b) There is more surface area across which synaptic vesicles can release acetylcholine. [1]
c) When action potentials reach the endplate, acetylcholine is released into the cleft. Calcium
ions enter the endplate from the cleft through channels. The acetylcholine diffuses across
the cleft to the sarcolemma of the muscle fibre, where it binds to specific receptors. This
causes the sarcolemma to be depolarised. Assuming the threshold value is reached, action
potentials spread over the muscle cell membrane and into the sarcoplasmic reticulum.
This causes calcium ions to be released from the sarcoplasmic reticulum. They attach to
troponin and allow the binding sites on actin to be exposed. [5]
d) A single action potential may not cause enough acetylcholine to be released to depolarise
the sarcolemma. But if successive action potentials each release some neurotransmitter,
it may build up sufficiently to exceed the threshold stimulus for the muscle fibre. [3]
4 a) Slow fibres use mainly aerobic respiration to make ATP so require an efficient delivery of
oxygen from blood in order to remain aerobic and make sufficient ATP for contraction. [2]
b) Any two other suitable differences, e.g. slow has large amount of myoglobin, fast has
very little myoglobin, slow resistant to fatigue, fast quickly becomes fatigued. [1]
c) Slow ATP hydrolysis means that the cycle of actinomyosin cross-bridge formation is
slower, so muscle contraction is slow. Jumping would require rapid contraction of limb
muscles which the slow loris is unable to do. [3]
d) At night, slow careful movement reduces the risk of falling from trees; slow movements
make the slow loris harder for predators to spot, they can remain active looking for
food for longer because they fatigue less quickly. [1]

Stretch and challenge questions


5 The difference in structure between these two types of muscle should be well described, and
the most important differences should be very clear, as well as mentioning their properties,
i.e. voluntary/involuntary control and the myogenic nature of cardiac muscle. These differences
should be related to the roles of the two types of muscle.

© Pauline Lowrie and Mark Smith 2015


18 Muscles and movement Answers

6 You should detail the position of dystrophin in muscle fibres and its role in linking actin to other
link proteins. This should be linked to the progressive symptoms in Duchenne muscular
dystrophy (DMD). You could also relate this to current research into dystrophin and possible
future treatments for DMD.

© Pauline Lowrie and Mark Smith 2015


19 Internal control Answers

Test yourself on prior knowledge (page 340)


1
Condition Why it is important to control this condition
Water content of the Maintenance of water potential of blood prevents osmotic
body damage to cells
Ion content of the body Contributes to water potential of blood (see above)
Temperature Maintains enzymes at their optimum temperature
Blood sugar Enables cells to respire and contributes to water potential of
the blood (see above)

2 Only the target cells contain the protein receptor molecule that is complementary in shape
to the hormone.

Test yourself (page 346)


1 A high glucose concentration lowers the water potential of the blood because more solute
molecules are present per unit volume.
2 This is when molecules move from a region of high concentration to a region of lower
concentration, across a membrane, using a carrier or channel protein.
3 Facilitated diffusion moves molecules down their concentration gradient whereas active
transport can move molecules against their concentration gradient. Facilitated diffusion
does not involve hydrolysis of ATP whereas active transport does.
4 This stops glucagon having a prolonged effect, so blood glucose concentration is not raised
too much.
5 This means that blood with a high glucose concentration first goes to the liver where excess
glucose is converted to glycogen, preventing a rise in the water potential of the circulating
blood. Also, it means that the b-cells in the islets of Langerhans are stimulated as soon as
the concentration of glucose in the blood goes up after a meal.
6 Each molecule of hormone can produce many molecules of cAMP and these in turn activate
large numbers of enzymes. This means that glucose concentration is raised very quickly.

Test yourself (page 347)


7 If insulin was taken orally it would be digested by protease enzymes in the gut.
8 Diet: eat regular, small meals; avoid eating high levels of sugar, which cause glucose levels
to increase rapidly; eat starch instead of sugars, as it releases glucose more slowly once
digested; eat plenty of fibre, which slows down the rate at which sugars are absorbed into
the bloodstream. Exercise: this uses up glucose in respiration and so reduces blood glucose
concentration.

© Pauline Lowrie and Mark Smith 2015


19 Internal control Answers

9 When blood glucose concentration is high, it lowers the water potential of the blood.
Therefore, water enters the blood from the body tissues by osmosis. This means there is a
greater volume of blood in the circulatory system, therefore blood pressure increases.

Required practical 11: Production of a dilution series of a


glucose solution and use of colorimetric techniques to produce
a calibration curve with which to identify the concentration of
glucose in an unknown ‘urine’ sample (page 349)
1
Concentration of Volume of 1 mol dm−3 Volume of distilled
sucrose/mol dm−3 required/cm3 water required/cm3
0.9 18 2
0.8 16 4
0.7 14 6
0.6 12 8
0.5 10 10
0.4 8 12
0.3 6 14
0.2 4 16
0.1 2 18

2 The filtered solution is blue. Red is the complementary colour to blue, so a blue solution
absorbs red light most strongly.
3 This makes sure the colorimeter is measuring the blue colour and nothing else. The blank
ensures that light absorbed by the water and the cuvette are ignored.
4 The student measures the percentage transmission for the two urine samples. She finds
these values on the y-axis of the calibration graph then draws a horizontal line to the curve.
Next, she draws a vertical line down to the x-axis. This gives the concentration of glucose in
the urine sample.

Test yourself (page 354)


10 The basement membrane normally prevents large proteins passing through. If the
basement membrane is damaged, large proteins are able to pass into the ultrafiltrate
because the gaps between the podocytes and the gaps in the capillary endothelium are too
large to keep them in.
11 Large surface area provided by microvilli and many mitochondria to provide ATP for active
transport.
12 Water moves from a region of high (less negative) water potential to a region of lower
(more negative) water potential. Reabsorption of glucose, salts and amino acids causes the
water potential of the ultrafiltrate inside the proximal convoluted tubule to increase above

© Pauline Lowrie and Mark Smith 2015


19 Internal control Answers

that of the epithelial cell, so water enters the cell down a water potential gradient. As the
water potential in the blood is lower than the water potential in the cell, the water enters
the blood by osmosis.
13 In a healthy person haemoglobin is only found inside red blood cells, and red blood cells are
too large to pass across the basement membrane.

Test yourself (page 357)


14 Water has been reabsorbed by osmosis in the proximal convoluted tubule, so equilibrium
has been reached.
15 The high salt concentration causes the tissues to have a lower (more negative) water
potential than the fluid in the descending tubule. Water leaves the descending tubule by
osmosis.
16 Sodium ions have a positive charge (they are cations) whereas chloride ions have a negative
charge (they are anions). As chloride ions diffuse, they create an electrochemical gradient;
the sodium ions diffuse down this electrochemical gradient.
17 A person working outside on a hot day is likely to lose a lot of water through sweating. If
they drink alcohol they will lose more water in their urine because ADH is inhibited. This
means they are at risk of dehydration, and they will not have enough water in their body to
produce enough sweat. If they fail to produce enough sweat, their body temperature could
rise to a dangerous level.
18 When the water potential of the blood falls below the normal level, a corrective mechanism
occurs (ADH release), which brings the water potential back to the normal level again.
Similarly, if blood water potential rises above the normal level, a corrective mechanism
occurs (ADH release is inhibited), which causes the water potential of the blood to return to
the normal level.

Practice questions (pages 358–9)


1 a) So blood glucose concentration was at its normal/lowest level; so blood glucose
concentration was not affected by the food/drink recently consumed. [1]
b) Patient X; higher blood glucose concentration at start/after fasting; blood glucose
concentration rises more and falls more slowly. [2]
c) i) Insulin released; increases permeability of liver and muscle cells to glucose;
stimulates respiration; stimulates conversion of glucose to glycogen. [3]
ii) Glucagon released; stimulates conversion of glycogen to glucose. [2]
2 a) Glucose and fructose [1]
b) i) Starch is digested/hydrolysed (by amylase) to glucose, which is absorbed into blood;
cellulose has β-glycosidic links/different shape and is not digested/hydrolysed. [3]

© Pauline Lowrie and Mark Smith 2015


19 Internal control Answers

ii) Glucose is absorbed directly into blood and raises blood glucose concentration
rapidly; lactose is digested to glucose and galactose and therefore has a smaller
effect on blood glucose. [3]
c) Glycogen is a storage carbohydrate/polysaccharide; it is formed from glucose by action
of insulin; glucagon is a hormone that stimulates conversion of glycogen to glucose. [5]
3 a) i) Ultrafiltration; pressure from pumping of heart; afferent vessel is wider than
efferent vessel; it forces water and small molecules out of capillary into renal capsule;
basement membrane prevents large molecules/proteins passing through. [4]
ii) Glucose reabsorbed in proximal convoluted tubule; by active transport; not enough
protein carriers to absorb all glucose in filtrate from people with diabetes. [3]
b) (Fewer sodium ions reabsorbed means) higher water potential in medulla; less steep
water potential gradient; less water reabsorbed from filtrate by osmosis; greater volume
of urine produced. [3]
4 a) Both show an increase in urea concentration for first 8 hours; shows urea being produced
but not excreted; as kidney function lost; urea concentration stays the same/drops slightly
in both when liver function stops. [3]
b) i) No liver function so urea is not made; no kidney function so urea is not excreted. [2]
ii) Liver function is lost so no more urea is made; kidney still functioning so urea is
gradually excreted. [2]

Stretch and challenge questions


5 The answer should include data from a few examples, related to the availability of water in the
environment. This should be accompanied by a clear explanation of how the loop of Henle is
involved in producing hypotonic urine.
6 This should involve an understanding that maintaining a stable body pH involves acid excretion.
7 Contrast negative feedback and positive feedback. Research hypothermia and hyperthermia, and
explain how these are examples of positive feedback. Use your knowledge of physiology to
justify the first aid treatments for these conditions.

© Pauline Lowrie and Mark Smith 2015


20 Genes, alleles and inheritance Answers

Test yourself on prior knowledge (page 361)


1 1B, 2E, 3H, 4I, 5J, 6G, 7K, 8A, 9F, 10C, 11D
2

3 Chromatids of a homologous pair cross over at chiasmata and exchange alleles. This produces
new combinations of maternal and paternal alleles.
4 This is when the homologous pairs of chromosomes separate to opposite poles of the cell
during meiosis independently of any other pair.
5

Test yourself (page 365)


1 Respiration in body cells releases heat as a by-product. Weak bonds that hold the enzyme in its
tertiary structure, such as hydrogen and ionic bonds, break, thereby changing the shape of the
enzyme’s active site.
2 A gene is a section of DNA located at a particular site on a DNA molecule, called its locus. The
base sequence of each gene carries the genetic code that determines the sequence of amino
acids during protein synthesis. An allele is a different form of a gene.

© Pauline Lowrie and Mark Smith 2015


20 Genes, alleles and inheritance Answers

3 Although it gives a person an increased chance of developing atherosclerosis, this does not
mean that the person will have a heart attack or a stroke, as by eating a healthy diet, avoiding
smoking and exercising regularly they may be able to prevent disease developing.

Test yourself (page 368)


4 Each body cell is diploid, i.e. the chromosomes are present in homologous pairs. There is a copy
of each gene on each chromosome in the homologous pair, so every body cell contains two
alleles of a gene, which may be the same or different.
5 There are only two chromosomes, so it is not possible to have more than two alleles of each
gene in one cell.
6 A large molecule, usually a protein, on the surface of a cell that stimulates the production of an
antibody.
7 IO because two copies of this allele are needed for it to be present in the phenotype.
8 a) A form of a gene with a (slightly) different base sequence.
b) IO is recessive because it only contributes to the phenotype when two copies are present. IA
and IB are codominant because both contribute to the phenotype in a heterozygote.

Test yourself (page 372)


9 a) The allele for short hair is dominant because both parents had short hair but some of the
offspring have long hair; therefore, the allele for long hair must be recessive.
b) Aa (any symbols are allowed as long as a heterozygous genotype is shown).
c)

© Pauline Lowrie and Mark Smith 2015


20 Genes, alleles and inheritance Answers

10 Both parents must be heterozygous to have a child with cystic fibrosis.


a)

b) 1 in 4 or 25%, as was the chance the first time they had a baby.

Test yourself (page 375)


11

12 a)

© Pauline Lowrie and Mark Smith 2015


20 Genes, alleles and inheritance Answers

b)

c)

Test yourself (page 377)


13

14 The Y chromosome is very small and carries very little genetic information.

© Pauline Lowrie and Mark Smith 2015


20 Genes, alleles and inheritance Answers

15 Only if her mother was a carrier and her father had haemophilia.

Test yourself (page 380)


16 a) Parents: WWDD, wwdd; offspring: WwDd
b)

© Pauline Lowrie and Mark Smith 2015


20 Genes, alleles and inheritance Answers

17 a)

b)

© Pauline Lowrie and Mark Smith 2015


20 Genes, alleles and inheritance Answers

18

Test yourself (page 382)


19 a)

Phenotype Possible genotypes

Agouti CACA, CACB, CACC

Black back yellow belly CBCB, CBCC

Black CCCC

b)

© Pauline Lowrie and Mark Smith 2015


20 Genes, alleles and inheritance Answers

20

Test yourself (page 384)


21 The gene for height and for fruit colour are very close together on the same chromosome, so
they tend to be inherited together. There are very few recombinants produced. Recombinants
are only produced when a cross-over occurs between the two gene loci.

© Pauline Lowrie and Mark Smith 2015


20 Genes, alleles and inheritance Answers

22 a)

b) The parental phenotypes are the two most common in the offspring. There are very few
recombinants (purple round, red long). This implies that the genes for flower colour and
pollen shape are linked on the same chromosome. The recombinants have been produced as
a result of crossing over between the two gene loci.

Practice questions (pages 385–6)


1 a) i) Two parents without Tay–Sachs have a child with Tay–Sachs; specific example,
e.g. 1 and 2 have 5, or 3 and 4 have 6. [2]
ii) 5 is a female with Tay–Sachs but father not affected; males have only one X
chromosome so he would have to have Tay–Sachs if it was sex-linked and his
daughter was affected. [2]
b) i) 4 = Aa
ii) 6 = AA or Aa
iii) 10 = AA or Aa [3]
2 a) A gene on the X or Y chromosome. [2]

© Pauline Lowrie and Mark Smith 2015


20 Genes, alleles and inheritance Answers

b) [4]

c) Male cats have only one X chromosome/tortoiseshell cats have two X chromosomes. [2]
3 a) None of these genotypes produces functional enzymes and substrate. [1]
b) i) AB, Ab, aB, ab [1]
ii) [2]

iii) 7/16 = 43.8 or 44% [1]

© Pauline Lowrie and Mark Smith 2015


20 Genes, alleles and inheritance Answers

4 a) i) A gene is a length of DNA that codes for a polypeptide/characteristic; an allele is a


different form of a gene. [1]
ii) Both alleles contribute to the phenotype in a heterozygote. [1]
b)
Individual Phenotype Genotype Explanation

2 Group B IBIO Has a child with group B but also has grandchild
with group O

5 Group A IAIO Has one child with group A and another with
group O

4 Group B IBIO Has one child with group AB and another of group
O

9 Group A IAIO Has group A but must carry recessive allele as one
parent is group O

[4]
5 a) TtCRCW (other symbols are acceptable but height gene is dominant/recessive and flower
colour is codominant) [1]
b) [3]

© Pauline Lowrie and Mark Smith 2015


20 Genes, alleles and inheritance Answers

c) Offspring all/almost all tall, pink; recombinants are only formed if/when crossing
over occurs. [2]
d) Cross with homozygous recessive (tt); if any dwarf offspring, plant is heterozygous;
include a diagram showing the crosses. [3]

Stretch and challenge questions


6 You should explain how genes that are closer together are more strongly linked, i.e. there are
fewer recombinants formed in genetic crosses because crossing over between the genes is less
likely when genes are very close together. A very good answer will express this mathematically,
for example explaining how cross-over values are calculated.
7 You should explain how the X and Y chromosomes determine sex in humans, and the role of SRY
on the Y chromosome. Then you should explain the chromosomal inheritance of the syndromes
mentioned and the similarities and differences between them. An excellent answer would
involve further research into other unusual chromosome combinations, e.g. XYY, or phenotypic
females who have inherited a Y chromosome with a non-functional SRY gene. You should relate
the geneotypes involving missing or additional chromosomes to non-disjunction during meiosis.
8 You should show that there are mechanisms other than the XY chromosomes that are found in
humans; for example the ZW system in birds and some reptiles, and the haplodiploidy system in
insects, whereby unfertilised eggs usually develop into males and fertilised eggs (which are
diploid) generally develop into females. You should also mention temperature-dependent
systems. An excellent answer will involve a discussion of current research and examine areas
where scientists are still unsure of the mechanism involved. You should also mention organisms
that change sex (for example clown fish) and relate this to specific hormone activity.

© Pauline Lowrie and Mark Smith 2015


21 Gene pools, selection and speciation Answers

Test yourself on prior knowledge (page 387)


1 A change in the base sequence of a gene that may result in a new allele.
2 This increases genetic diversity in the zoo population.
3 a) This is because, in natural conditions, the populations would not interbreed as they are on
separate islands.
b) This is because they are trying to maintain an ‘insurance’ population that might, one day, be
returned into the wild. There is no point in breeding animals that would not occur naturally.

Test yourself (page 392)


1 No, because many of these conditions are not met. In particular, the population is subjected to
a lot of migration in and out; but also mating is not random. It is possible to argue that some of
the other conditions of the Hardy–Weinberg equation do not apply as well.
2 The frequency of non-rollers, q2 = 210/700 = 0.3; q = √0.3 = 0.55; p + q = 1, therefore
p = 1 − 0.55 = 0.45; heterozygotes = 2pq = 2 × 0.55 × 0.45 = 0.5; therefore 0.5 × 700 people are
heterozygous = 350.
3 a) Non-agouti is recessive, therefore q2 = 0.16; q = √0.16 = 0.4; p + q = 1;
therefore p = 1 − 0.4 = 0.6. Therefore, the frequency of the dominant allele is 0.6 and the
frequency of the recessive allele is 0.4.
b) p2 = homozygous dominant = 0.6 × 0.6 = 0.36. Therefore 36% of the population is
homozygous dominant. Heterozygotes = 2pq = 2 × 0.6 × 0.4 = 0.48. Therefore 48% of the
population is heterozygous.
4 108 have normal hair, therefore q2 = 108/1200 = 0.09, so 9% is homozygous recessive.
q = √0.09 = 0.3; p = 1 − q = 1 − 0.3 = 0.7; p2 = 0.72 = 0.49. Therefore 49% is homozygous for
woolly hair.
Heterozygotes = 2pq = 2 × 0.7 × 0.3 = 0.42. Therefore, the frequency of people heterozygous for
woolly hair is 0.42.
5 3200 people are Rhesus negative, therefore q2 = 3200/20 000 = 0.16; therefore q = √0.16 = 0.4;
p = 1 − q = 1 − 0.4 = 0.6; heterozygotes = 2pq = 2 × 0.6 × 0.4 = 0.48; 0.48 × 20 000 = 9600 people.

Test yourself (page 395)


6 Yellow, as this will be better camouflaged against the grass.
7 There is a smaller spread between the extreme ends of the curve and the mean. (Note that the
mean and the mode are the same value in these examples.)

© Pauline Lowrie and Mark Smith 2015


21 Gene pools, selection and speciation Answers

8 The NHS has provided good medical care to everyone, so even babies that are born very small
are more likely to survive. Also, very large babies are more likely to survive through good
antenatal and perinatal care, availability of Caesarean section, etc.

Test yourself (page 399)


9 a) Genus
b) Species
10 The environment on each island is slightly different. Therefore, the characteristics that make an
organism more likely to survive on one island will not be the same as those on another island. In
other words, each island has a different selection pressure. The allele frequencies will differ on
each island, and over a very long period of time sufficient differences will accumulate in each
population that they will become reproductively isolated and therefore new species.
11 Haploid = having one set of chromosomes; diploid = having two sets of chromosomes; polyploid
= having several sets of chromosomes (usually refers to three or more sets).
12 There are many examples here. The main source is mutation, but variation then exists as a
result of crossing-over and independent segregation during meiosis, and random fertilisation.
13 Examples include food supply, predation, disease and camouflage, but there are many other
possible answers.

Test yourself (page 401)


14 The role of chance has a much bigger effect in a small population as one or two individuals
represents a high proportion of the alleles. In a large population, a few individuals will not make
a significant difference.
15 a) The single woman with the disease had a lot of descendants, so this has increased the
frequency of the allele in the gene pool. It is an isolated area, so her descendants are likely to
have remained in the area.
b) It will have a negligible effect on the allele frequency since people with the condition have
passed on the allele for Huntington’s before they develop symptoms.

Practice questions (pages 402–3)


1 a) 84/233 are homozygous recessive. Therefore
q2 = 84/233 = 0.36;
q = √0.36 = 0.6,
p = 1 − 0.6 = 0.4;
2pq = 2 × 0.6 × 0.4 = 0.48;
48% of population are heterozygous. [3]

© Pauline Lowrie and Mark Smith 2015


21 Gene pools, selection and speciation Answers

b) Any two from: no migration, large population, random mating, no selection against any
phenotype, no mutation [2]
2 a) The gene pool is all the alleles of all the genes in a population. [1]
b) Two, because cell is diploid/one allele on each chromosome. [1]
c) 32/200 are black, which is recessive, therefore q2 = 32/200 = 0.16 and:
i) q = √0.16 = 0.4 [1]
ii) p = 1 − 0.4 = 0.6 [1]
3 a) Variation in ancestral population; all in same environment/no geographical isolation;
mutation/example, e.g. polyploidy; reproductive isolation/example; unable to produce
fertile offspring. [4]
b) i) Genus
ii) Species [1]
4 a) A species is a population that can interbreed to produce fertile offspring. [1]
b) Variation present (in ancestral population); geographical isolation; mutation; idea of
different selection pressures in different environments; change in allele frequency;
reproductive isolation; new species when no longer able to interbreed. [4]
c) Compared base sequence with that of other species of dolphin; found number of
differences in base sequences; calculated time of divergence by assuming constant rate
of mutation. [3]
5 a) i) For comparison/as a control; to show Anolis leg length is unchanged in absence of
larger lizard. [2]
ii) Disruptive, because both extremes are selected for. [1]
b) It would be unethical to introduce a foreign species that could establish itself, and which
may become a pest/wipe out native species; it could also introduce disease. [3]
c) Yes, because longer-legged lizards: would be more likely to die in flooding; are less able
to climb trees; will not pass on alleles (accept converse for shorter-legged lizards).
OR
Yes, because: more medium-length legs in population; predator lizards wiped out;
therefore less selection for leg length. [2]

Stretch and challenge questions


6 You should recognise that, if one phenotype is selected against, either (p2 + 2pq) will increase in
the next generation, or q2 will increase. Therefore the value of p and q will vary each generation.
You may research modifications of the formula that take account of selection, e.g. by including a
selection coefficient. You could attempt to derive a selection coefficient for yourself.
7 You may research this. There are a few theories to explain this. All the theories suggest that the
heterozygotes are at an advantage at surviving a potentially fatal infectious disease that has

© Pauline Lowrie and Mark Smith 2015


21 Gene pools, selection and speciation Answers

been common among white Europeans in the past, e.g. cholera, tuberculosis or typhoid. Even if
students don’t carry out research they should be able to make some kind of sensible suggestion
for the heterozygote having a selective advantage over both homozygotes.

© Pauline Lowrie and Mark Smith 2015


22 Populations in ecosystems Answers

Test yourself on prior knowledge (page 404)


1 You place a measuring tape in a straight line across the area to be sampled. You then sample the
organisms along the tape. This can be done by recording every organism touching the tape, or
by sampling at regular intervals along the tape by using a quadrat.
2

Term Definition
Median The middle value in a series of numbers.
Mean A calculated ‘central’ value of a set of numbers. To calculate it, you add up all the
numbers, then divide by how many numbers there are.
Mode The number which appears most often in a set of numbers.

3 By cultivating only one kind of crop/farm animal over a large area; getting rid of weeds or pests
by using pesticides or cultural methods; getting rid of trees or hedgerows to make fields larger.

Test yourself (page 407)


1 Some birds may be finding food while the count is taken; young birds are not included; some
birds may not be visible when the count is taken, e.g. in a rock crevice.
2 1F, 2E, 3B, 4C, 5D, 6A

Activity: Intraspecific competition (page 408)


1 1 hectare = 10 000 m2 so there were 6.7 seeds sown per m2.
2 If the sowing density is lower, the cotton plants grow taller. This may be because there is less
intraspecific competition for light, nutrients, etc.
3 It does not have a significant effect on height of the cotton plants because the standard
deviations overlap.
4 Standard deviation tells you how dispersed the data are from the mean.
5 This would give them a larger sample so the results would be more precise and therefore
repeatable.

Test yourself (page 412)


3 It is a biotic factor because it is a factor related to another population, i.e. to do with the living
component of the ecosystem.
4 Increasing the sowing density increases the grain yield up to about 100–200 seeds m−2. After
this point the grain yield decreases.
5 The optimal sowing density is about 100 seeds m−2. Although a slightly higher sowing density
gives a similar yield, it is not cost-effective to sow more seeds than necessary. A higher sowing

© Pauline Lowrie and Mark Smith 2015


22 Populations in ecosystems Answers

density gives a lower biomass of weeds, but gives a lower grain yield so this is not a sensible
decision.
6 In oak woodlands, both species feed on the same food source: acorns.

Required practical 12: Investigation into the effect of a named


environmental factor on the distribution of a given species
(pages 415–6)
1 Series of kite diagrams, similar to Figure 9.11, showing the data in Table 9.3, set out as in Figure
9.12.
2 There are various possible answers here, e.g. how far into the ground you can push a tent peg,
or place a plastic measuring cylinder that has had its base cut off on the ground and time how
long it takes for a measured volume of water to percolate into the ground.
3 There might be some species close to the transect line that don’t get sampled, but would be
present if a quadrat square was used. Also, a pin on the quadrat might touch a tiny plant that
doesn’t actually cover a large area, but this converts to 10% cover in the data. A quadrat square
allows assessment of percentage cover with divisions that are less than 10. One of the benefits
of the point quadrat, however, is that it is easy to use and convert the data to percentage cover.
4 Plant cover in general is reduced when trampled, but it appears that dandelions are much less
able to survive trampling than plantain.
5 There is mechanical damage to plants by tissues being crushed and cells being damaged. In
addition, the soil becomes compacted, which reduces air spaces in the soil, so the roots have
less oxygen available for respiration. Plantain has a bud lower in the soil than dandelion, so it is
less likely to be damaged. Its leaves are tougher than dandelion leaves and it grows in a flatter
rosette.

Activity: Investigating the mark-release-recapture method of


analysing population size (page 418)
1 There is no right answer, but if several groups in a class do the exercise you are likely to find one
or two fairly accurate estimates but also one or two that are not very accurate at all. Discuss the
overall reliability of this method of estimating population size with your classmates.
2 There is no right answer here, but generally the mean of 10 samples is more accurate.
3 Carrying out sampling 10 days in a row has practical difficulties as well as other issues, e.g.
migration, births/deaths in that time, animals habituating to traps so either avoiding them or
seeking them out, etc.
4 The point here is to think about other variables and to model what happens if that variable
changes. In many cases you will find the estimate is better if one of these variables is altered!

© Pauline Lowrie and Mark Smith 2015


22 Populations in ecosystems Answers

5 You should realise that the equation does not provide complete accuracy but it can give a fairly
reliable estimate. You should also appreciate the many difficulties in estimating the population
size of highly mobile small mammals with a short life span.

Test yourself (page 418)


7 C
8 136; round your answer to the nearest whole number as you cannot have a fraction of a fish.
9 They could record the DNA fingerprints of all the whales sampled in a specific period of time.
These would be regarded as the ‘marked and released’ sample. After a suitable time interval,
they could take another sample. Any whales in the second sample whose genetic fingerprints
were already recorded are counted as marked, and the population can be estimated from this.
10 No animals die or are born between taking the first sample and the second sample; marking
does not harm the animal in any way, e.g. making it more vulnerable to predators; the marked
and released animals mix freely with the rest of the population; all animals are equally likely to
be caught; there is no immigration or emigration from the population.

Test yourself (page 423)


11 The plants that normally grow on chalk grasslands are adapted to low-nitrate concentrations.
Adding nitrogen-containing fertilisers allows other species to grow, outcompeting the typical
chalk land species.
12 Ploughing, which breaks up roots of plants; animal grazing, which prevents taller plants growing;
harvesting, which removes the plants from the field; burning; weeding.
13 Seeds remain in soil after the fire; they may also be blown in by wind or dropped into the area
by birds (e.g. in droppings).

Practice questions (pages 424–5)


1 a) Species richness is the number of different species in an area. [1]
b) i) Increases until about 350 m then reduces; trend not exact/some variation in values. [1]
ii) Conditions are hostile at front of dunes because of salt-laden winds, sand as a
substrate, which is unstable and is very poor in nutrients and does not hold water,
so fewer plants can survive; further back, conditions/named condition better so more
plants are able to grow; at rear conditions are better because sheltered from wind,
organic material (humus) has accumulated in sand, so it is becoming soil. The soil can
hold more water and contains more nutrients. Species are larger, therefore fewer in
a given area/more woody species which outcompete smaller species for light. [3]

© Pauline Lowrie and Mark Smith 2015


22 Populations in ecosystems Answers

c) It would increase, because there are more food sources; more habitats/greater
stratification. [2]
2 a) Climax community [1]
b) New species colonise the area and outcompete species that were already there. [2]
b) i) The sheep eat plants so that woody/taller species are unable to grow.
ii) Ploughing breaks up roots so that plants cannot become established/plants
are removed. [2]
3 a) An ecological niche is the abiotic and biotic conditions that organisms in a population
must have in order to survive and reproduce/a description of all the environmental factors
that limit the growth of a given population. [1]
b) 617; based on:
total number in sample
population size = number of marked animals in population × [2]
number of marked animals in sample

c) Some may have died/new animals may have been born/there may have been migration/
the dye might have worn off. [2]
4 a) i) A population is all the members of a species living in one area at the same time.
ii) A community is all the populations of all the species that live in one area at the
same time. [2]
b) 596; based on:
total number in sample
population size = number of marked animals in population × [2]
number of marked animals in sample

c) Either divide field into a grid of squares 1 m2/suitable size plus use a method of choosing
squares at random (e.g. random coordinates), count buttercups in each square and multiply
up to get an estimate of number of plants in whole field; or place quadrats at random on
field using random coordinates, count number of buttercups in each and multiply up to
get estimate of number of plants in whole field. [3]
5 a) To show that both species can survive in these conditions [1]
b) Interspecific competition for food/algae; P. aurelia is a better competitor/outcompetes
P. caudatum. [2]
c) Two suitable conditions, e.g. temperature/volume of growth medium/composition of
growth medium/concentration of algae at start/concentration of Paramecium at start. [1]
d) Can investigate competition while other variables controlled; therefore fewer/no
confounding variables; small so occupies little space; in a ‘real-life’ situation other factors
are present/there are predators; cannot be certain that competition would be similar in
environmental conditions. [4]
6 a) Prey mite increases in number when predator low because there is less predation/falls
when predator high because there is more predation; when prey population is high,

© Pauline Lowrie and Mark Smith 2015


22 Populations in ecosystems Answers

predator population increases because there is less (intraspecific) competition for food; there
is a time lag in increase in predators due to time for reproduction. [3]
number at week 24 - number at week 10
b) × 100% [1]
number at week 10

c) Two from: small, so many organisms can be studied in a small space; simple nutritional
requirements; can control confounding variables/no other organisms present. [2]

Stretch and challenge question


7 Suitable investigations might be the effect of soil compaction on the distribution of plantain
(e.g. comparing an area by the goal mouth of a football pitch with a similar area near the edge
of the football pitch) or the distribution of dandelions on two grass verges with different aspects.
Credit is given for random sampling, suitable number of quadrats, measuring confounding
variables, choice of a suitable statistical test and relating the results to biological knowledge.

© Pauline Lowrie and Mark Smith 2015


23 The control of gene expression Answers

Test yourself on prior knowledge (pages 426–7)


1 Both contain phosphate; both contain a five-carbon sugar; both contain adenine, guanine and
cytosine; RNA has a ribose sugar, DNA has a deoxyribose sugar; RNA contains uracil instead of
thymine.
2 GCCAUGCU
3 CGFBEAHJDI
4 A polypeptide contains several/many amino acids but the protein may contain one or more
polypeptide chains.
5 It makes the molecule more stable, and can be used to correct errors during DNA replication, as
the second strand is complementary to the template strand.

Test yourself (page 429)


1 There is a tiny risk that a virus might be present in the mouse cells that shows no symptoms in
the mouse, but that might cause harm if it infects human cells.
2 a) They differentiate to form specialised cells to replace damaged/dead cells.
b) Advantages: divide quickly; they are pluripotent. Disadvantages: using embryos can be seen
as unethical; genetically different from cells of recipient; may divide too much and cause
cancer.

Test yourself (page 431)


3 a) Because any codon starting with GC codes for alanine. (See Table 4.6 on page 70.)
b) The base sequence is read in threes. When a base is deleted, all the codons change from the
point of the deletion.
c) A base substitution changes only one codon (so Ile is coded for instead of Tyr). But this does
not change the next codon or any others following it.
4 a) The haemoglobin gene is not expressed in a white blood cell.
b) i) A gamete contains the genes that are passed on to offspring.
ii) It may lead to uncontrolled cell division, i.e. cancer.
5 Collect data on the number of people diagnosed with cancer and the distance they live from a
mobile phone aerial. Plot a graph of incidence of cancer (y axis) and distance from an aerial (x
axis). Apply a statistical test to determine the probability of any correlation being due to chance.

Test yourself (page 434)


6 It can pass through the phospholipid bilayer.
7 Oestrogen receptors are found in several different cell types.

© Pauline Lowrie and Mark Smith 2015


23 The control of gene expression Answers

8 Cost of drug compared to increase in life expectancy; whether the patient has other serious
health issues; age of patient; whether patient has dependent family members.

Test yourself (page 438)


9 Those inheriting the defective chromosome from the mother do not develop Prader–Willi
syndrome.
10 Paternal chromosomes/genes are epigenetically imprinted differently from maternal
genes/chromosomes.
11 a) The plant only flowers when it is spring or summer/when weather is warm.
b) By adding methyl/removing acetyl groups.

Activity: An investigation into tumour formation in transgenic


mice (page 439)
12 Clones are organisms that are genetically identical.
13 An oncogene is a mutated proto-oncogene. Proto-oncogenes control cell division so that it only
occurs when needed for growth or repair. Oncogenes cause cells to divide too frequently.
14 This was so that the only differences between the mice were the oncogenes they contained, so
they would know that the age at which they developed tumours was due only to the oncogenes
they carried and not any other genes.
15 20 days
16 The ras oncogene starts to cause tumours after 20 days but myc only causes tumours after 105
days.
17 The effects of myc and ras were added together because both genes were active. The mice
developed tumours as early as the mice that only had the ras gene, which shows the effect of
the ras gene, but far more mice developed tumours, which indicates a cumulative effect of both
genes.

Practice questions (pages 442–3)


1 a) Epigenetics is the inherited changes in DNA that do not involve a change in
base sequence. [2]
b) Methylation is adding a methyl group; to cytosine bases; to histone protein;
represses transcription. [3]
c) Any two, e.g. diet, smoking, exposure to radiation, exposure to chemicals. [1]
d) Removing methyl groups/epigenetic tags; using an enzyme. [2]
2 a) Pluripotent describes cells that can (divide) to produce many kinds of cell; but not
all kinds. [1]

© Pauline Lowrie and Mark Smith 2015


23 The control of gene expression Answers

b) Cells are specialised/differentiated; cannot divide; would not produce eye cells. [2]
c) The implanted cells have the same DNA as the woman’s other/body cells; will not cause
an immune response; embryonic cells are different genetically; embryonic cells mean
embryo destruction: some people think this is unethical. [4]
d) To make sure there are no side effects/harm; to ensure it is effective; cells might develop
into wrong/unintended cell type; could divide to form cancer cells. [4]
3 a) Monoclonal antibodies are identical antibodies; to a specific antigen. [2]
b) Binds to HER-2 receptors; stops growth factor binding. [2]
c) Only effective against HER-2 receptors; some kinds of breast cancer don’t have
these receptors. [2]
d) Protein; would be digested/broken down in gut. [2]
4 a) Binds to mRNA with complementary base sequence; the mRNA is cut into two; by
dicer/enzyme/protein complex; cut mRNA cannot make proteins. [4]
b) i) Same as spray with drug/siRNA in; but without the drug. [2]
ii) The virus is harmful to children/not harmful to adults; adults can report side effects. [2]
c) The virus could be particularly harmful to people who have had transplants; taking
immunosuppressive drugs; so immune system not working against virus. [2]

Stretch and challenge questions


5 You ought to be able to describe the inheritance of these two syndromes. For a good answer
you can then give a coherent comparison of the two inheritance patterns, while recognising
similarities. An excellent answer will draw on current research to offer explanations for the
inheritance patterns.
6 You should be able to explain the inheritance of tortoiseshell coat colour relatively easily as this
is covered in sex linkage. However, for a better answer you can explain the inheritance pattern
on the basis of Barr body formation in female nuclei and the fact that this is random. This should
be linked to the fact that Carbon Copy does not resemble the cat from which she was cloned.

© Pauline Lowrie and Mark Smith 2015


24 Gene cloning and gene transfer Answers

Test yourself on prior knowledge (page 444)


1 The code for each amino acid is the same in all organisms. This is shown by the fact that
transcription and translation are the same in all living organisms. Scientists were able to predict
that genetic engineering should work, and when it was tried experimentally, their prediction
was confirmed.
2 A base sequence of DNA that codes for a polypeptide or functional RNA.
3 Triplet (codon is acceptable)

Test yourself (page 448)


1 TGC GCT AGT ACT
2 A short piece of single-stranded DNA complementary to the mRNA.
3 It makes a single-stranded DNA copy of the viral RNA.
4 7
5 Because the phosphate groups in each nucleotide are negatively charged.
6 So that the DNA separates at a regular rate and its passage is not blocked by debris or a change
in density.

Test yourself (page 450)


7 a) The enzyme works and does not denature in the hot environment in which the bacterium
lives.
b) The enzyme remains active and does not denature despite being heated up every cycle.
8 This means it is more likely that a single-stranded piece of DNA will bind to a primer than to
another single-stranded piece of DNA.

Test yourself (page 455)


9 They are smaller and less dense than the rest of the bacterial DNA.
10

Enzyme Process
Restriction enzyme Cuts DNA at a specific base sequence
Reverse transcriptase Makes a single-stranded DNA copy of an RNA sequence
(DNA) ligase Joins a new piece of DNA into a plasmid

Test yourself (page 458)


11 a) cDNA is a complementary copy of the mRNA, so the introns have been spliced out.

© Pauline Lowrie and Mark Smith 2015


24 Gene cloning and gene transfer Answers

b) Yeast is eukaryotic, so it contains endoplasmic reticulum and Golgi bodies to modify and
secrete the protein.
12 Pollen grains do not contain chloroplasts, so they are very unlikely to contain the foreign genes.

Test yourself (page 459)


13 This is because inserting a copy of the dominant allele will counteract the effect of the recessive
allele. If the disease was caused by a dominant allele, the dominant allele would need to be
removed or disrupted in some way.
14 A gene for a characteristic considered desirable but not affecting health could be added; for
example, a gene for hair colour, eye colour, intelligence, musical ability, etc.
15 The gene for the human protein can be inserted into a zygote along with a promoter gene that
switches the human protein gene on only in milk-producing cells.

Practice questions (pages 461–2)


1 a) i) To insert the gene into the target/goat cells [1]
ii) To cut DNA; at a specific base sequence/recognition site [2]
b) i) So it is copied into all/most cells in the adult; before the cells
specialise/differentiate [2]
ii) DNA unzips; one strand becomes a template; RNA nucleotides align by
complementary base pairing; joined by RNA polymerase; mRNA leaves nucleus/binds
to ribosome; tRNA brings specific amino acids; codon/anticodon binding; peptide
bonds formed between amino acids. [6]
c) Agree: don’t know whether this causes long-term harm to the animal; could be produced
by different method.
Disagree: protein in milk, therefore animal does not suffer; medical uses of protein bring
benefits; animals are not treated any worse than other farm animals. [4]
2 a)

Item Description

Plasmid The term used for the small circular piece of DNA

Sticky end(s) The name for the structure labelled A

Restriction (enzyme) The enzyme used to cut the DNA

(DNA) ligase The enzyme used to attach the human gene to the circular DNA
[4]
b) A marker gene is a gene introduced alongside the target gene that allows detection of
cells that have taken up the target gene. [1]

© Pauline Lowrie and Mark Smith 2015


24 Gene cloning and gene transfer Answers

c) i) Any two valid points, e.g. bacteria have no membrane-bound organelles/no


nucleus/circular rather than linear DNA/cell wall of peptidoglycan [2]
ii) Glycoproteins made by Golgi apparatus/proteins are modified at Golgi;
bacteria/prokaryotes do not have Golgi apparatus. [2]
3 a) (DNA) polymerase (enzyme) and nucleotides [1]
b) One for beginning and one for end of section/one for starting point on each strand [1]
c) 64 (26 is acceptable) [1]
d) Makes DNA single-stranded/separates DNA strands; by breaking hydrogen bonds [1]
4 a) Primers bind to Sarcosystis genes; but not to other DNA. [1]
b) i) It joins nucleotides together to make a complementary strand. [1]
ii) It is not denatured during heating; works for many cycles/no need to keep adding
fresh enzyme. [2]
5 a) Use of vector/named vector; cut DNA with restriction enzyme; sticky ends; insert using
ligase; reference to marker gene. [4]
b) Allows development of sexually mature females; to breed fresh supply of males
for release. [2]
c) Reduces the fruit fly population so greater crop yields; specific to this species; avoids use of
insecticides which might harm useful insects/accumulate in food chains; risk of gene being
passed to other insects; could damage populations of useful/pollinating insects. [4]

Stretch and challenge question


6 A good answer should include different uses of GM, e.g. in agriculture, medicine and research.
You should use good scientific terminology, avoiding journalistic expressions such as ‘playing
God’. You should be able to distinguish concerns that have some background in science from
those that are based on ignorance. A good answer will use a range of proper examples, and
should be able to argue whether genetic engineering is a useful tool in that specific case.

© Pauline Lowrie and Mark Smith 2015


25 Using genetic technology Answers

Test yourself on prior knowledge (page 463)


1 They are negatively charged because of the phosphate groups.
2 DNA polymerase
3 To mark the beginning and end of the sequence to copy, and for attachment of DNA
polymerase.

Test yourself (page 465)


1 For: it allowed scientists all over the world to have access to the information rapidly, so that any
scientific advances made as a result would be available sooner; any medical treatments
developed using the information would be cheaper as they would not have to pay to use the
HGP information.
Against: this meant that the HGP had to attract public funding, e.g. via charities or government
funding.
2 If the gene mutated and the mutation was disadvantageous it would be unlikely to survive.
Therefore, if there are many variants in the population these have been selected for by natural
selection. If the protein is for an antigen on the malaria parasite, it will be an advantage to the
parasite to have many variations of this protein. This would mean that humans may become
resistant to one variant but would be unlikely to develop resistance to all the variants.

Test yourself (page 469)


3 These are likely to be antigens. A host may undergo a mutation that gives them resistance to
this antigen, e.g. the mutation may give a different shaped receptor protein on their cells that
the antigen cannot fit into. However, a mutation may occur in a pathogen that changes the
shape of the antigen so that it can infect hosts with resistance to the first antigen. These
pathogens survive, reproduce and pass on their alleles. In this way, there is an on-going
selection pressure on genes that code for antigens.
4 They are synthesised in a ‘gene machine’: see Chapter 11.
5 So that the DNA probe can bind to its complementary sequence if it is present. If the DNA is
double-stranded there are no exposed bases for the probe to bind to.
6 There could be another (rare) mutation that has a different base sequence, so the probe does
not bind.

Activity: Breast cancer and ovarian cancer (pages 470–1)


1 They are tumour-suppressor genes because they regulate cell growth.
2 They are dominant.

© Pauline Lowrie and Mark Smith 2015


25 Using genetic technology Answers

3 It is difficult to predict the exact probability of developing cancer for one individual, but the risk
for the population in general is a known figure.
4 A mutant BRCA allele increases the chances of developing breast cancer by age 50 by at least 16
times and by age 70 by at least eight times. It also increases the chance of developing ovarian
cancer by age 70 by at least 14 times.
5 There is more breast tissue than ovarian tissue. Also, women may have had ovaries removed in
hysterectomy.

Test yourself (page 471)


7 No right or wrong answers here, and you may come up with many different points. You may
argue that screening for some conditions may enable people to make sensible lifestyle choices
to reduce the chances of the disease developing, e.g. if you are predisposed to get cancer or
heart disease you can eat a healthy diet and refrain from smoking and thereby reduce the
chances of the disease developing. It also enables people to choose whether to have children, as
they may not want to pass on a harmful allele. On the other hand, screening people for a
disease that is incurable and develops later in life may not be helpful as there is nothing a
person can do with the information, except to become depressed. It might also lead to a person
having a higher quotation for life insurance than other people.
8 For: men can also get breast cancer; these genes also predispose to other kinds of cancer; men
may pass on these genes to their children. Against: men are less likely to get breast cancer than
women; screening causes stress and worry.

Test yourself (page 475)


9 So they can be detected as DNA bands on the gel or membrane are too small to see.

Practice questions (pages 476–7)


1 a) i) A vaccine is an (injection of) antigens, which cause antibodies/memory B cells to
be made. [1]
ii) Virus contains/carries antigens from all three viruses; these antigens stimulate
T-helper cells; activate B-cells; divide to form (three different) clones of plasma
cells; these produce specific antibodies against the antigens; memory cells
are produced. [5]
b) Virus enters cells; replicates; produces more effective immune response. [2]
2 a) Extract DNA/make more copies using PCR; cut with restriction enzyme; carry out gel
electrophoresis; transfer to nylon membrane/Southern blot; make single-stranded;
add radioactive/fluorescent probe. [6]

© Pauline Lowrie and Mark Smith 2015


25 Using genetic technology Answers

b) Suspect 2; all the bands match crime-scene DNA. [2]


c) In favour: saves time/effort when solving crimes; might deter criminals if they know
they can be traced.
Against: there could be an innocent explanation for the DNA being found there; could be
confused with a person with a similar DNA fingerprint; invasion of privacy/concerns if data
passes to other people; people might be able to find further information about a person
from stored DNA.
(Must have at least one argument on each side for full marks.) [4]
3 a) i) Use of primers; specific to the base sequence at start of gene [2]
ii) To estimate length of unknown DNA fragments; similar-length fragments travel
similar distances [2]
iii) Band at the bottom left of the gel, lane 1. [1]
b) Only one band; so DNA is not cut by the restriction enzyme [2]
c) Two bands; DNA cut by restriction enzyme [2]
d) Carrier/has one copy of mutated allele and one normal allele; three bands each
matching one of the other two samples [2]

Stretch and challenge question


4 You should be aware that sequencing a child’s genome will not be especially informative at the
present time, as we have not elucidated the proteome, but nevertheless many harmful genes
can be identified. It is worth noting that technology will improve considerably during the early
life of a child born today. There is therefore the possibility of making decisions to reduce the
health risks to the child. A good answer will discuss the implications this may have for life
insurance, and that carrying a specific harmful allele does not always mean that it will have a
harmful effect on the individual. You might also reflect on poor lifestyle choices people might
make if they are told they do not have an increased risk of developing a disease such as heart
disease or cancer.

© Pauline Lowrie and Mark Smith 2015

You might also like